Sie sind auf Seite 1von 92

Physics 21

Fall, 2011

Equation Sheet

speed of light in vacuo


Gravitational constant
Avogadros Number
Boltzmanns constant
charge on electron
free space permittivity
free space permeability
gravitational acceleration

F2 on 1 =

3.00 108 m/s


6.67 1011 N m2 /kg2
6.02 1023 mol1
1.38 1023 J/K
1.60 1019 C
8.85 1012 C2 /(N m2 )
4 107 T m/A
9.807 m/s2

c
G
NA
kB
e
0
0
g

1 q1 q2 (r1 r2 )
40 |r1 r2 |3

; Eplane =
r
20
for  plate
0 capacitor
A
A
Q = CV ; C = 0 K = 
d
d
2
Q
2
Ucap = 12 CV = 12
C
Uind = 12 LI 2
L
V = IR
R=
A
P = IV
P = I 2R
1
20
Q
E=
=
0 A

1 dQ (r r )
40 |r r |3

E = V


V
V
V
= i
+ j
+k
x
y
z
V f Vi =

f

E dl

1 Q
dQ
1
; dV =
40 r
40 |r r |
1
uelec = 12 0 E 2 , umag =
B2
20
V=

Work =

R = mv /(qB)




A B = 


F dl


E dA =

Q
0

d
dt

E dl =




u du
=
a2 + u 2

a2 + u 2

du
1
= tan1
a2 + u 2
a

v=

T /

 
u
a

ln a2 + u

1
2

(T =tension)

v = (347.4 m/s)

T /300

v = f = /k
= 2f

k = 2/

T = 1/f

(T =period)

P  = 12 A2 2 v

1
C

RC time constant = RC
LR time constant = L/R
Q(t) for RLC circuit
Q0 exp(Rt/2L) cos t
2 =

= sin cos

1
R2

LC
4L2

cos sin

E dA

u du
=
a2 + u 2

XR = R, XL = L, XC =

solenoid B = 0 nI
solenoid L = 0 N 2 A/l

C = 2r
C = d
A = r2
A = 4r2
V = 43 r3

circumference of circle
circumference of circle
area of circle
surface area of sphere
volume of sphere

cos(a b) = cos a cos b sin a sin b


du

= ln u + a2 + u2
2
2
a +u

)
2

long wire: B =

eective = 1 + 2

= cos

B dA

d
B dl = 0 I + 0 0
dt

sin(

0 I
2R
center loop: B = 0 I/2R
dQ
I=
I = neAvd
dt
Vs
Ns
Is
Np
=
,
=
Vp
Np
Ip
Ns

m =
1
0
=B
= IA

i =Ci (parallel) or Ri (series):

sin(a b) = sin a cos b cos a sin b

B dA = 0

F = qv B; dF = Idl B
0 Idl (r r )
dB =
4
|r r |3

1
1
1
=
+
eective
1
2

circ. orbit

j

k

Ay Az 
By Bz 

i
Ax
Bx

6.626 1034 J s
1.055 1034 J s
9.11 1031 kg
1.6726 1027 kg
1.6749 1027 kg
1.6605 1027 kg
8.99 109 N m2 /C2

h
h
= h/2
me
mp
mn
u
k

i =Ci (series) or Ri (parallel):

Eline =

F = qE
dE (at r) =

Plancks constant
Plancks constant/(2)
electron rest mass
proton rest mass
neutron rest mass
atomic mass unit
1/(40 )

ab
a+b
sin a + sin b = 2 cos
sin
2
2





du
u
=
(a2 + u2 )3/2
a2 a2 + u 2

du
= ln u
u
2

cos2 d =
0

2D
1 2D
= 2
2
x
v t2
1
S=
(E B)
0

c = 1/ 0 0

c 2
S=
=
B0
0
E0 B0
Erms Brms
=
=
20
0

= h/p

1
2

1
un+1
n+1

du
1
= ln(a + bu)
a + bu
b

1
du = eau
a

ln u du = u ln u u

1
 cE02
2 0

un du =

u du
1
=
(a2 + u2 )3/2
a2 + u 2
au

ax2 + bx + c = 0

b b2 4ac
x=
2a

EBv
xp >
h

sin2 d =
0

KE = p2 /(2M )
(plane wave)

p=
hk

(
h = h/2)

ei = cos + i sin

E=
h = hf

(de Broglie)

2 2
h

= i
h
2M x2
t

August 16, 2011

Physics 21
Fall, 2011

Solution to HW-2

21-13 Three point charges are arranged on a line. Charge


q3 = +5.00 nC and is at the origin. Charge q2 = 3.00 nC
and is at x2 = 4.50 cm. Charge q1 is at x1 = 1.00 cm. What
is q1 (magnitude and sign) if the net force on q3 is zero?
q3

q1

q2

x3=0

x1

x2

We can work this problem without using vectors by thinking it through. Since q2 and q3 have opposite sign, the force
on q3 exerted by q2 is attractive (towards the right). If the
total force on q3 is to be zero, the force exerted by q1 must
be repulsive (toward the left). Thus q1 and q3 must have the
same sign, and q1 must be positive.
We can find the magnitude of q1 by equating the magnitude of the forces on q3 exerted by q1 and q2 :
1 |q1 q3 |
1 |q2 q3 |
=
2
40 x1
40 x22
Cancelling like terms on both sides of the equation, we find
 2

2
x1
1.0 cm
|q2 | =
(3 nC) = 0.148 nC.
|q1 | =
x2
4.5 cm
We already concluded that q1 was positive.
A more general way to solve this problem is to use the
vector expressions for the Coulomb force. We want
0 = F1 on 3 + F2 on 3 ,
where
0=

1 q2 q3 (r3 r1 )
1 q1 q3 (r3 r1 )
+
.
40 |r3 r1 |3
40 |r3 r2 |3

and we can evaluate the forces using the locations of the


charges. Because q1 is at the origin, r3 = 0. Also, r1 = x1i
and r2 = x2i. Substituting for the vectors gives


1
q1 q3 (0 x1 )i q2 q3 (0 x2 )i
0=
+
40
|0 x1 |3
|x2 |3


q2 x2
q3 q1 x1
+
=
i
40 |x1 |2
|x2 |2

21-15 Three point charges are located on the positive x


axis of a coordinate system. Charge q1 = 1.0 nC is 2.0 cm
from the origin, charge q2 = 4.0 nC is 4.0 cm from the
origin and charge q3 = 6.0 nC is located at the origin. What
is the net force (magnitude and direction) on charge q1 =
1.0 nC exerted by the other two charges?
This problem is very similar to 21-13, and the same diagram applies. Here we need the sum F of F2 on 1 and F3 on 1 ,
which is


q1 q2 (r1 r2 ) q1 q3 (r1 r3 )
1
+
F=
,
40
|r1 r2 |3
|r1 r3 |3
where, as before, r3 = 0, r1 = x1i, and r2 = x2i. Then


1
q2 (x1 x2 )i q3 (x1 x3 )i
F=
q1
+
40
|x1 x2 |3
|x1 x3 |3


q2 (0.02 m) q3 (0.02 m)
1
q1
+
=
i
40
(0.02 m)3
(0.02 m)3
Substituting the other numbers leads to


4 nC(.02 m) 6 nC(.02 m)
9
F = (9 10 )(1 nC)
+
i
(.02 m)3
(.02 m)3
= 2.25 104 N i

21-11 In an experiment in space, one proton is held fixed


and another proton is released from rest a distance d away.
What is the initial acceleration of the proton after it is released?
From Physics 11 you know that F = ma, or a = F/m.
So just find the electrostatic force on one proton due to the
other proton, and then divide by the mass. Well drop the
vector notation and just find the magnitude:
a=

e2
1
40 mp d2


2
1.602 1019 C
= 9 10 N m /C
(1.67 1026 kg) d2


When you substitute for d, dont forget to convert to meters.


For d = 3 mm = 0.003 m, the result is
2

a = 1.54 104 m/s .

Note that the terms in the denominators are lengths and


must be positive. The quantity in brackets must be zero, so
we obtain

2
 3
x2  x1 
4.5  1.0 
q1 =   q2 =
(3.0 nC) = 0.148 nC
x1 x2
1.0  4.5 
This formula agrees with the previous result. Because we
were careful with the signs, the formula gives the correct
answer for any combination of signs of the three charges and
for the two vector components x1 and x2 . (We took x3 = 0.)
August 31, 2011

21-46 Two particles having charges q1 = 0.600 nC and


q2 = 5.00 nC are separated by a distance of d = 1.60 m. At
what point along the line connecting the two charges is the
total electric field due to the two charges equal to zero?

q2

q1
0

Since both charges are positive, its easy to keep track of


the direction of the electric field. The field at x from q1
points to the right, and the one from q2 points to the left.
These two fields must be equal in magnitude for their vector
sum to be zero. Therefore
q2
1
1 q1
=
2
40 x
40 (x d)2
Cancelling the common factor of 1/(40 ), we can rewrite
the above equation as

(d x)2
q2
q2
dx
=
=

2
x
q1
x
q1

YF 21-50 mod A point charge q1 = 4.00 nC is at the


point x = 0.60 m, y = 0.80 m, and a second point charge
q2 = +6.00 nC is at the point x = 0.60 m, y = 0. (a,b)
Calculate the x and y components of the net electric field
at the origin due to these two point charges. (c,d) Calculate
the x and y components of the net electric field at the point
x = 0.90 m, y = 0.40 m due to these two point charges.
Use the vector expression given in class for the field E at
r due to a charge Q at point r . Apply this formula to get
the field at r due to Q1 ; apply it again to get the field at r
due to Q2 , and then add the results (superposition).
E (at r) =

Remember, r = field point; r = charge point.


y

Q1 = -4.0 nC
(0.6, 0.8)

(0.9, 0.4)
Q2 = +6.0 nC
(0.6, 0.0)
x

Solving for x, we find


x=

d

1 + q2 /q1

Substituting the specific numbers given above leads to

(a,b) Find E at origin, r = 0i+0j. For Q1 , r = 0.6i+0.8j, so


r r = 0.6i 0.8j and |r r | = 1.0 m. For Q2 , r = 0.6i,
so r r = 0.6i and |r r | = 0.6 m.

x = 0.412 m.
Note that instead of taking the square root and solving
a linear equation for x, one could also set up a quadratic
equation. One must identify the correct root of the quadratic
equation, but the result is the same.

1 Q (r r )
40 |r r |3

4 nC(.6 i .8 j)m 6 nC(.6 i)m


+
(1.0 m)3
(.6 m)3
 



3.2
nC
1 2.4
3.6
=

i
+ j
40
13
(.6)3
13
m2


nC
1
14.3 i + 3.2 j
=
40
m2


= 128.3 i + 28.77 j N/C

1
E=
40

(c,d) Find E at point r = 0.9 i+0.4 j. For Q1 , r = 0.6i+0.8j,


so rr = 0.3 i0.4 j and |r r | = 0.5 m. For Q2 , r = 0.6i,
so r r = 0.3 i + 0.4 j, and |r r | = 0.5 m.


4 nC(.3 i .4 j)m 6 nC(.3 i + .4 j)m


+
(0.5 m)3
(0.5 m)3


i(1.2 + 1.8) + j(1.6 + 2.4) nC
1
=
40
0.125
m2
nC
1
4.8 i + 32 j
=
4
m2
 0

= 43.2 i + 287.7 j N/C

1
E=
40

Physics 21
Fall, 2011

Solution to HW-3

21-96 Positive charge Q is uniformly distributed around


a semicircle of radius a. Find the electric field (magnitude
and direction) at the center of curvature P .

dQ = ds
= ad

From this result we see the E has a magnitude of


Q
2 2 0 a2
and points in the y direction or downward. The x component of the field is zero, as one would expect from symmetry.
21-105 Three charges are placed as shown in the figure.
The magnitude of q1 is 2.00 C, but its sign and the value of
the charge q2 are not known. Charge q3 is +4.00 C, and the
net force F on q3 is entirely in the negative x-direction. a)
Calculate the magnitude of q2 . b) Determine the magnitude
of the net force F on q3 .

d
x

We start with the equation from the equation sheet that


gives the field at the field point r in terms of a charge dQ at
the charge point r :
dE(r) =

1 dQ (r r )
40 |r r |3

The field point r (where we want to know E) is at the origin,


so r = 0, and the charge point (where dQ is located) is
r = a cos i + a sin j,
where is the angle above the x-axis, so
r r = a cos i a sin j,

and

|r r | = a.

Since we want to add up all the contribution from all dQ,


we must relate dQ to d so that we can integrate over the
angle spanned by the semicircle. The arc length ds swept
out by an angle d is ad, so the charge dQ on ds is
dQ = ds = a d,
where is the linear charge density (charge per unit length):
=

Q
.
a

With all these substitutions, the original equation becomes


1 ( a ad)(a cos i a sin j)
dE =
40
a3
Q
(cos i + sin j)d.
=
4 2 0 a2
Q

We can find the total field at the origin by integrating:


Z
Z
Q
E = dE =
(cos i + sin j) d.
2
2
0 4 0 a
We will look at each component of the integral over separately. We see that
Z
Z
sin d = 2,
cos d = 0 and
0

so we get
E=

Q
4 2 0 a2

(0i + 2j) =

Q
2 2 0 a2

j.

(a) We first determine the sign of the charge q1 . We can do


this by thinking about which direction the force will be in
for the different combinations of signs for charges q1 and q2 .
Since there is no y-component of the force on q3 we know
that q1 and q2 must have opposite signs. Since the force is
directed in the negative x-direction we can infer that q1 must
be negative and q2 must be positive.
To determine q2 we calculate the the total force F = F1 on 3 +
F2 on 3 on q3 . Expressions for F1 on 3 and F2 on 3 follow from
the general expression for F2 on 1 on the equation sheet:
1 q3 q2 (r3 r2 )
1 q3 q1 (r3 r1 )
,
F2 on 3
.
F1 on 3 =
3
40 |r3 r1 |
40 |r3 r2 |3

We need the position vectors of each charge. Let q1 be at


the origin. Then the position vectors r1 and r2 of q1 and q2
are trivial. For r3 , we notice that cos = 4/5 = x/4 where
x is the x-component of the position vector of q3 . Along
with this and Pythagoreans theorem we can find both x
and y-components of r3 . The result is
r1 = 0,
r2 = 5 cm i,
r3 = 3.2 cm i + 2.4 cm j

Knowing these position vectors we can write F1 on 3 and


F2 on 3 , in terms of the unknown charge q2 :
F1 on 3 = 36 N i 27 N j
F2 on 3 = (24 106 N/C) q2 i + (32 106 N/C) q2 j

Since the net force on q3 is in the negative x-direction, the


sum of the y-components must be 0. From the sum of the
y-components we find that the charge q2 = +0.844 C. The
charge is positive, as expected by the reasoning above.
(b) Using q2 , we can now determine the net force on q3
by adding the components together. We already know that
there is no net y-component, so the net force has only an xcomponent, which is the sum of the x components of F1 on 3
and F2 on 3 . The magnitude of the total force is the absolute
value of its x component, |F| = 56.26 N.

September 10, 2011

21-87 A proton with the mass m is projected into a uniform electric field that points vertically upward and has magnitude E. The initial velocity of the proton has a magnitude
v0 and is directed at an angle below the horizontal. (a)
Find the maximum distance hmax that the proton descends
vertically below its initial elevation. You can ignore gravitational forces. (b) After what horizontal distance d does the
proton return to its original elevation? (c) Find the numerical value of hmax if E = 520 N/C, v0 = 5.00 104 m/s, and
= 35.0 . (d) Find the numerical value of d if E = 520 N/C,
v0 = 5.00 104 m/s, and = 35.0 .
Because the proton is a charged particle (charge e), when
it enters a region of uniform electric field, it experiences a
constant force according to the relationship F = eE. In
previous physics classes, you have studied how a constant
force influences the motion of an object. In particular, recall
that Newtons Second Law tells us how force and acceleration
are related through the equation F = ma. Combining these
two equations allows us to calculate the acceleration:
a=

eE
.
m

Since the electric field is uniform, the acceleration will be


constant, and the proton will follow a parabolic trajectory,
as shown in the figure.

The acceleration of the proton will be in the same direction


as the electric field. Thus, there is zero acceleration along
the x direction. This leads to the same type of problem
as that studied in projectile motion near the surface of the
earth. The only difference is that the acceleration is upward
instead of downward. The equations we need to use are:
x direction
x = x0 + vx0 t
vx = vx 0 = constant

However, height is a scalar quantity, so we can solve for


y1 and take the absolute value. We get an answer of
hmax =

mv02 sin2
.
2eE

(b) In order to calculate x2 , the position when the proton


comes back up to its original height, we need to find the
time it takes to get there, t2 . Because of the symmetry of
the motion, t2 is just twice the time t1 needed to reach the
peak. Using vy = vy0 + at, we substitute in vy = 0 at t1 ,
use our expression for vy0 , and solve for t1 . Doubling this
result gives us
t2 =

2v0 sin
.
a

Substitute this expression into x = x0 + vx0 t, use our expression for a, and set x0 = 0 to get
x2 =

2mv02 sin cos


.
eE

This expression is also the answer for d, the distance travelled


in the x direction, since d = x2 x0 = x2 0.
(c) and (d) Substituting in the given values, along with the
fundamental constants e = 1.602 1019 C and m = 1.673
1027 kg (see the equation sheet), yields
hmax = 8.26 103 m

and d = 4.72 102 m.

21-99 Two 1.20 m nonconducting wires meet at a right


angle. One segment carries 2.00 C of charge distributed
uniformly along its length, and the other carries 2.00 C
distributed uniformly along it, as shown in the figure. Find
(a) the magnitude and (b) the direction of the electric field
these wires produce at point P , which is 60.0 cm from each
wire. If an electron is released at P , what is (c) the magnitude and (d) the direction of the net force that these wires
exert on it?

y direction
y = y0 + vy 0 t + 12 at2
vy = vy 0 + at
2
+ 2ay
vy2 = vy0

Before we go any further, we must resolve the intial velocity


vector into x and y components:
v0 = vx0 i + vy0 j,
where
vx0 = v0 cos

and vy0 = v0 sin .

(a) When the proton reaches its maximum height, the y


component of the velocity reaches 0 (see v1 in the figure).
2
Substituting into vy2 = vy0
+ 2ay yields 0 = v02 sin2 +
2ay1 , where y1 is the position at maximum height. By the
choice of coordinate system, this will be a negative position.

(a) In class we worked out the vector electric field due to a


finite line of charge at a field point located directly outward
from the midpoint of the line. The magnitude of the field is
E=

1
a

,
20 d d2 + a2

where d is the distance of the point from the line of charge,


and a is half the length of the line. In this exercise, the two
wires have the same length, the same magnitude of charge,

and are the same distance from the point P . Thus, we can
adapt the formula above to the charge and spatial orientation
of each wire by putting in the correct direction for each field.
Recalling that = Q/L and a = L/2, we find
E=

1
1 Q
p
40 d d2 + (L/2)2

= (9 109 Nm2 /C2 )


= 35355N/C.

2 106 C
1
p
0.6m
(0.6 m)2 + (0.6 m)2

The field E due to the negatively charged wire is directed


to the left, while the field E+ due to the positively charged
wire is straight down. We can use superposition to find the
net electric field
Enet
Enet

= E + E+ = 35355 N/C i 35355 N/C j


q
2 + E 2 = 5.00 104 N/C
= E
+

(b) Since the field vectors are perpendicular and of equal


magnitude, the net electric field will be 45 from either vector, and 135 counterclockwise from the +y axis.
(c) The force acting on a charge q is F = qE, so at P the
magnitude of the force on the electron is
Fnet = eEnet = (1.60 1019 C)(5.00 104 N/C)
= 8.00 1015 N.

(d) Since the electron has a negative charge, the force will
be directed opposite (180 ) from the electric field. Counterclockwise from the +y axis, = 315 .
21-104 A thin disk with a circular hole at its center,
called an annulus, has inner radius R1 and outer radius
R2 . The disk has a uniform positive surface charge density on its surface. (a) Determine the total electric charge
on the annulus. (b) The annulus lies in the yz -plane,
with its center at the origin. For an arbitrary point on
the x -axis (the axis of the annulus), find the magnitude
of the electric field E. Consider points above the annulus in the figure. (c) Find the direction of the electric
field E. Consider points above the annulus in the figure.

surface area. For an annulus, A = R22 R12 . Mastering


Physics is expecting a symbolic expression, so simply enter

R22 R12

(b) This problem is similar to Example 21.12 in the textbook.


Our target is the electric field along a symmetry axis of a
continuous charge distribution. We can represent the charge
distribution as a collection of concentric rings of charge dQ.
From lecture, we know the field of a single ring on its axis
of symmetry, so all we have to do is add the contributions
of the rings. For a single ring,
Ering = i

Qx
1
40 (x2 + a2 )3/2

where Q is the total charge on the ring, x is the distance


along the axis from the ring to the point at which we are
finding the field, and a is the radius of the ring. We will now
assume an infinitesimally thin ring, with charge dQ on it, of
radius r , and thickness dr . It will have a contribution to
the electric field
1
dQ x
dE = i
40 (x2 + r2 )3/2
Before we can integrate this, we need to come up with an
expression that tells us how much charge is on our infinitesimally thin ring. We will approximate the area of the ring
as circumference thickness (this approximation works because the ring is infinitesimally thin). So, dQ = dA =
2r dr . We now have
r dr
x
dE = i
20 (x2 + r2 )3/2
To get the net electric field, we integrate this expression from
r = R1 to r = R2 .
R2

Z
x R2
r dr
i x 1
E = i
=
20 R1 (x2 + r2 )3/2
20
x2 + r2 R1

1

1

q
q
= i
20
2
2
1 + (R1 /x)
1 + (R2 /x)
Note that the integral used here is on the table of integrals
on the equation sheet. Mastering Physics is only asking for
the magnitude of the electric field here, so enter


1
1

q
q
20
2
2
1 + (R1 /x)
1 + (R2 /x)
(c) Here we select the direction of the electric field that was
found in part (b), the positive x -direction.

(a) Given a uniform positive surface charge density , the


total electric charge on any surface is A, where A is the

Physics 21
Fall, 2011

Solution to HW-4

22-7 The electric eld due to an innite line of charge is


perpendicular to the line and has magnitude E = /20 r.
Consider an imaginary cylinder with a radius of r = 0.200 m
and length l = 0.465 m that has an innite line of positive
charge running along its axis. The charge per unit length
on the line is = 7.15C/ m. (a) What is the electric ux
through the cylinder due to this innite line of charge? (b)
What is the ux through the cylinder if its radius is increased
to r = 0.515 m? c) What is the ux through the cylinder if
its length is increased to l = 0.760 m?

22-10 A point charge q1 = 4.00 nC is located on the x-axis


at x = 2.00 m, and a second point charge q2 = 6.00 nC
is on the y-axis at y = 1.00 m. What is the total electric
ux due to these two point charges through a spherical surface centered at the origin and with radius (a) 0.500 m, (b)
1.50 m, (c) 2.50 m?

q2
Sa

q1
Sb
Sc

The problem asks for total electric ux through a surface.


Gausss Law helps here. The Equation Sheet gives Gausss
Law in the form:

Q
E dA = ,
0

(a) Because the electric eld lines are perpendicular to the


line of charge, they will also be perpendicular to the surface
of the curved or barrel part of the cylinder, and parallel
to the ends of the cylinder. Because the line of charge is
concentric with the cylinder, the eld strength will also be
constant along the barrel part of the cylinder. Then the
electric ux can just be calculated by:
= EAbarrel =

2rl = 375, 700 Nm2 /C


20 r

Note that the rs cancel; the ux is independent of the radius of the cylinder. We can also use Gauss law to do the
calculation and obtain the same expression.
=

where the left hand side is the total electric ux E through


a surface S, and Q is the charge enclosed by S.
(a) Surface Sa , whose radius is 0.500 m, encloses no charge.
The total electric ux is zero (in any units).
(b) Surface Sb encloses q2 but not q1 , so Q = q2 :
E =

q2
6.00 109 C
= 678 NC1 m2 .
=
0
8.85 1012 C2 Nm 2

Note the units: NC1 is electric eld and m2 is area.


(c) Surface Sc encloses both charges, so Q = q1 + q2 :
q1 + q2
4.00 109 6.00 109
=
0
8.85 1012
= 226 NC1 m2 .

E =

l
Qenc
=
= 375, 700 Nm2 /C
0
0

(b) We already found in (a) that the ux is independent of


r; Gauss law also tells us that the ux through the cylinder
only depends on the charge enclosed. Because increasing
the radius does not change how much charge is enclosed, the
answer for part (b) should be the same as the answer for
part (a).
= 375, 700 Nm2 /C
(c) If the length of the cylinder is increased, the new ux
must be calculated because now more charge on the line is
enclosed.
=

l
Qenclosed
=
= 614, 000 Nm2 /C
0
0

September 12, 2011

22-20 The electric eld 0.400m from a very long uniform


line of charge is 840 N/C. How much charge is contatined in
a 2.00-cm section of the line?
The equation for the electric eld of an innite wire is
E=

1
,
20 r

22-24 A point charge of 2.11 C is located in the center


of a spherical cavity of radius rcav = 6.50 cm inside an insulating spherical charged solid. The charge density in the
solid is = 7.36 104 C/m3 . Calculate (a) the magnitude
and (b) the direction of the electric eld inside the solid at
a distance r = 9.48 cm from the center of the cavity.

where r is the perpendicular distance to the wire, and is


charge per unit length. Solving for gives
= 20 rE
= 2 8.8541 1012 0.4 840 = 1.86 108

C
m

If we muliply (in C/m) by the length (in m) we get charge:


0.02 = 3.74 1010 C
22-26 A conductor with an inner cavity, like that shown
below, carries a total charge of Qcond = +4.80 nC. An additional charge within the cavity, insulated from the conductor,
is q = 6.20 nC. (a) How much charge is on the inner surface of the conductor? (b) How much charge is on the outer
surface of the conductor?

(a) The Gaussian surface G shown includes the inner surface


of the conductor. The
 electric eld must be zero inside the
conductor, making G E dA = 0, and thus the total electric
charge Qencl enclosed by G must be zero. The charges contained within the surface are the charge in the cavity q and
the charge on the conductors inner surface qinner , so
Qencl = 0 = qinner + q

qinner = q = 6.20 nC.

(b) The total charge on the conductor Qcond is the sum of


the charge on the inner surface qinner and the charge on the
outer surface qouter . Hence
Qcond = 4.8 nC = qinner + qouter
qouter = 4.8 nC qinner = 4.8 nC 6.20 nC = 1.4 nC

(a) To nd the electric eld we can use Gauss Law,



Qencl
E dA =
,
0
S
where we choose the Gaussian surface S to be a sphere of
radius r = 9.48 cm centered at the point charge. At every
point on S the vector dA and E are perpendicular to S and
are therefore parallel (or antiparallel well provisionally
assume the former). By symmetry, the electric eld has the
same magnitude E everywhere on the surface, so we can
evaluate the surface integral in terms of the unknown E,


Qencl
E dA =
E dA = 4r2 E =
.
0
S
S
All that remains is to determine the charge enclosed by the
Gaussian surface, which consists of the point charge qpoint
and the charge Qinsul on the insulator, which we can calculate as V . (Note that the charge on an insulator can be
evenly distributed throughout its volume with density .)
The volume V of the insulator inside S is the volume of S
less the volume of the cavity,
4
4 3 4 3
3
r rcav = (r3 rcav
)
3
3
3

4 
= (.0948 m)3 (.0650 m)3 = .00242 m3 .
3

V =

Thus the total charge enclosed is given by


Qencl = qpoint + Qinsul = qpoint + Vencl
= 2.11106 C+(.000736 C/m3 )(.00242 m3 )
= 3.3 107 C.
Now we can use the relation between E and Qencl :
E=
=

Qencl
4r2 0
4(9.48

102

3.3 107 C
m)2 (8.854 1012 C2 /(Nm2 ))

= 3.3 105 N/C.


(b) Since the magnitude E must be positive, the minus sign
we obtained for E above tells us that E and dA must be
antiparallel, that is, E points into the surface.

Physics 21
Fall, 2011

Solution to HW-5

22-4 A cube has sides of length L = 0.330m. It is placed


with one corner at the origin as shown in the figure. The
electric field is not uniform but is given by

For surface S5 , the normal direction is in the +x direction,


so we integrate the i component over the surface.
5 =

4.41x dy dz = 4.41xL2

Evaluating this at x = L, gives the flux:

E = [4.41 N/(C m)] x i + [3.29 N/(C m] z k.

5 = 4.41L3 = 0.158 N m2 /C.

(a) Find the electric flux through each of the six cube faces
S1 , S2 , S3 , S4 , S5 , S6 . (b) Find the total electric charge inside
the cube.

Surface S6 is similar, but the surface is located at x = 0 so


the flux evaluates to 6 = 0.
(b) To find the total electric charge inside the cube, we can
apply Gauss Law, which tells us that the total flux through
this cube is equal to the charge enclosed, divided by 0 .
1 + 2 + 3 + 4 + 5 + 6 =

Qencl
0

Solving for Qencl and substituting the numbers gives


Qencl = (0 + 0.118 + 0 + 0 0.158 + 0)0
= 3.54 1013 C.

R
(a) The electric flux is defined as = E dA. Because
dA is always perpendicular to the surface, we only need the
component of the field that is normal to the surface of interest. (The dot product of dA with the parallel component
of the field will be zero.) For each of the six surfaces, it will
work out that the component normal to the surface is constant, so the surface integral will just be the constant value
of the normal component times the surface area L2 .
For surface S1 , there is no component of the electric field
normal to the surface (in the y direction j), so the electric
flux 1 = 0.
For surface S2 , the component of the electric field normal

to the surface is in the +z direction, so take only the k


component of E. The integral over the surface is
Z

3.29z dx dy = 3.29zL2 .

To evaluate the electric field at the surface we set z = L,


and then the flux is
2 = 3.29L3 = 0.118 N m2 /C
Surface S3 is similar to S1 ; there is no electric field component in the normal direction j, so the flux 3 = 0.
For surface S4 , the normal direction is in the z direction,
component. The
so we need to take the negative of the k
flux is that component integrated over the surface.
4 =

22-26 A conductor with an inner cavity, like that shown


below, carries a total charge of qtot = +5.80 nC. The charge
within the cavity, insulated from the conductor, is qcenter =
7.10 nC. (a) How much charge is on the inner surface of
the conductor? (b) How much charge is on the outer surface
of the conductor?
qouter
qinner

qtot = +5.80 nC
Gaussian
Surface

qcenter
-7.10 nC

(a) The insulated point in the center of the cavity will induce an opposite charge on the inner surface of the conductor. The induced charge will have the same magnitude as
the center charge, qinner = +7.10 nC. We can prove this by
placing a Gaussian surface within the conductor that encompasses the entire cavity and point charge. Inside the
conductor E = 0, and therefor Qencl = qcenter + qinner = 0.
(b) The total charge qtot on the conductor is a constant and
does not change, but we must find how it is divided between
the inner and the outer surface.
qtot = qinner + qouter = qouter = qtot qinner
qouter = 5.80 nC 7.10 nC = 1.30 nC

3.29z dx dy = 3.29zL2

But evaluating the electric field at z = 0 results in 4 = 0.

September 15, 2011

23-8 Three equal 1.20C point charges are placed at the


corners of an equilateral triangle whose sides are 0.500 m
long. What is the potential energy of the system? (Take as
zero the potential energy of the three charges when they are
infinitely far apart.)
The total potential energy is obtained by summing up the
potential energy of interaction between each pair of charges
(superposition). If we label the three charges 1, 2, and 3
then
U = U12 + U13 + U23

q1 q3
q2 q3
1 q1 q2
(
+
+
)
=
40 r12
r13
r23

Since all three charges are identical and the separation of


any pair is the same, then
U=

3(1.2 106 C)2


1 3q 2
= (9 109 )
= 0.078 J
40 r
0.500m

23-5 A small metal sphere, carrying a net charge of q1 =


2.90 C, is held in a stationary position by insulating supports. A second small metal sphere, with a net charge of
q2 = 7.80 C and mass 1.80 g, is projected toward q1 .
When the two spheres are ri = 0.800 m apart, q2 is moving
toward q1 with speed vi = 22.0 m/s. Assume that the two
spheres can be treated as point charges. You can ignore the
force of gravity. (a) What is the speed of q2 when the spheres
are rf = 0.430 m apart? (b) How close does q2 get to q1 ?

For part (a) we are given rf , and we need to solve Eq. (1)
for vf . The result is
vf = 15.5 m/s.
For part (b), we know that when q2 is closest to q1 , the KE
reaches its minimum value of zero, so we must set vf = 0 in
Eq. (1) and solve for rf . The result is
rf = 0.295 m.
23-79 Electric charge is distributed uniformly along a thin
rod of length a, with total charge Q. Take the potential to
be zero at infinity. (a) Find the potential at the point P ,
a distance x to the right of the rod. (b) Find the potential
at the point R, a distance y above the right-hand end of
the rod. (c) In part a, what does your result reduce to as x
becomes much larger than a? (d) In part b, what does your
result reduce to as y becomes much larger than a?

We will express the potential dV at points P and R from


charge dQ at the charge point r = x i, and then we integrate
over the length of the charged rod (from x = a to x = 0).
The general formula is
dQ
1
.
dV =
40 |r r |
The charge dQ can be related to the length dx using the
linear charge density:
Q
dQ = dx = dx .
a

(a) For point P , r = x i and r r = (x x ) i. Hence

For this problem we will use the conservation of energy. We


first notice that the two charges are both negative, so the
Coulomb force is repulsive. Particle 2 must therefore start
with positive kinetic energy KE, and as it approaches particle
1, KE will diminish and the potential energy U will increase
by the same amount. We know
KE = 21 mv 2

and

U=

1
q1 q2
,
40 |r1 r2 |

1 q1 q2
= 0.689841 J.
40 ri

Since energy is conserved, Etot has the same value at any


other value of rf :
Etot = 12 mvf2 +

1 q1 q2
.
40 rf

By equating the two expressions for Etot , we see


2
1
2 mvf

1 q1 q2
= 0.689841 J.
40 rf

V =

dV =

1 Q
40 a

dx
1 Q x+a
=
ln
x x
40 a
x

(b) For point R, r = y j and r r = x i + y j. Hence


dx
1 Q
p
40 a (x )2 + y 2
Z
Q
dx
y
1 Q 0
p
=
ln p
V=
2
40 a a (x )2 + y 2
40 a
a + y2 a

dV =

where |r1 r2 | is the distance between the charges. We can


evaluate the total energy Etot by substituting the given initial values ri and vi :
Etot = 21 mvi2 +

1 Q dx
40 a (x x )

dV =

(1)

Another expression for the argument of the log follows from


p
y
a2 + y 2 + a
p
=
y
a2 + y 2 a
(c,d) For x or y large compared to a, we can consider the
whole charge Q on the line to be a point charge at the origin. The potential is then 1/(4) times Q/x for c or times
Q/y for d. Mathematically the result follows if we use the
approximation ln(1 + ) for 1. The key steps are
x+a
a
a
ln
= ln(1 + )
x px
px

a
a2 + y 2 + a
y2 + a
a
.
ln
ln 1 +
ln
y
y
y
y

Physics 21
Fall, 2011

Solution to HW-6

23-9 A point charge q1 = 4.10 nC is placed at the origin,


and a second point charge q2 = 2.9 nC is placed on the
x-axis at x = +21.0 cm. A third point charge q3 = 2.1 nC
is to be placed on the x-axis between q1 and q2 . (Take as
zero the potential energy of the three charges when they are
infinitely far apart.) a) What is the potential energy of the
system of the three charges if q3 is placed at x = +11.0 cm?
b) Where should q3 be placed to make the potential energy
of the system equal to zero?
(a) The general formula for the potential energy between any
two point charges labeled 1 and 2 is
U12 =

1
q1 q2
,
40 |r1 r2 |

where |r1 r2 | is the distance between the two charges. To


find the total potential energy of several charges, we must
find the potential energy due to each pair of point charges.
So the total potential energy here is
Utotal = U12 + U13 + U23

q1 q3
q2 q3
q1 q2
1
+
+
=
40 |0 .21 m| |0 .11 m| |.21 m .11 m|
= 3.53 107 J

(b) We can say that q3 is placed at some point x between


the other two charges and determine which value of x gives
the system a total potential energy of zero. Incorporating
this into the expression for the total potential energy gives:

q1 q3
q2 q3
q1 q2
1
=0
+
+
Utotal =
40 .21 m
x
(.21 m x)
We can save ourselves some trouble by eliminating several
overall factors that wont affect the solution x. Since all the
terms have exactly the same units, we can drop the conversion factors to SI units and write distances in cm and charges
in nC. The result is
(4.1)(2.9) (4.1)(2.1) (2.9)(2.1)
+
+
=0
21
x
(21 x)
Multiply through by x(21 x) to get a quadratic equation:

23-31 (a) An electron is to be accelerated from a velocity


of 2.50 106 m/s to a velocity of 8.00 106 m/s. Through
what potential difference must the electron pass to accomplish this? (b) Through what potential difference must the
electron pass if it is to be slowed from 8.00 106 m/s to a
halt?

This problem is a conservation of energy problem. The only


forms of energy involved are electrical potential energy (U )
and kinetic energy (K). Note that potential difference
means the change in voltage (V2 V1 ) from the initial value
V1 to the final value V2 , and that the relationship U = qV
is not on your equation sheet.
(a)
K1 + U1
2
1
2 mv1

= K2 + U2

+ qV1 = 21 mv22 + qV2

q(V2 V1 ) = 21 m(v22 v12 )


The final equation shows that the change in potential energy
is the negative of the change in kinetic energy, as expected.
Dividing by the charge gives
V2 V1 =

m 2
(v v12 ).
2q 2

Using the velocities given and remembering that q = e for


an electron gives us the answer
V2 V1 = 164 V.
Think carefully about the sign. A positively charged particle
would slow down when the potential goes up (like a mass
going uphill), but an electron behaves in the opposite way.
(b) With the same equation we found for part (a), we can
recalculate for the new velocities that
V2 V1 = 182 V.
In this case, the particle slows down, so a positively charged
particle would go through a positive change in potential, but
again the electron does the opposite.
Note that on Mastering Physics, the question asks for the
value of V1 V2 , not V2 V1 . So, the numerical answers you
should give are 164 V and 182 V.

0.5662x2 26.59x + 180.81 = 0


Using the quadratic formula we find x = 38.72 or 8.25. Remember that these values are in centimeters. Since the first
solution is not between the two other point charges, we can
ignore it. So q3 must be placed in between the two charges
at x = 8.25 cm to give the system a total potential energy
of zero.

September 16, 2011

23-36 A very long insulating cylinder of charge of radius


3.00 cm carries a uniform linear density of 15.0 nC/m. If you
put one probe of a voltmeter at the surface, how far from the
surface must the other probe be placed so that the voltmeter
reads 175 V?
The symmetry of such a uniformly charged cylinder is the
same as the symmetry of a uniform line of charge. In fact,
outside of the cylinder, the electric field is the same as though
all the charges were concentrated on a line along its axis. (We
can see this by applying Gausss law to a Gaussian surface
like that used for the uniform line of charge.) This electric
1
where
field is listed on the equation sheet as Eline = 2
0 r
is the linear charge density and r is the distance away from
the line. The electric field lines are directed radially outward
from the surface of the cylinder.
Now that we know what the electric field looks like, we can
calculate the potential difference (the quantity measured by
the voltmeter) by evaluating the line integral of the field
between the probes of the voltmeter. We will evaluate
Z f
E dl
Vf Vi =

From the second equation, Ey = 0 when x = C/A. Then


(from the first equation) Ex = 0 when
Ay + 2B(C/A) = 0

Be careful with the signs. The argument of the logarithm is


greater than one, so the logarithm is positive. Thus V is
negative. That makes sense; the potential decreases as one
goes further away from the positively charged cylinder.
Now, we solve for d and evaluate.

20 V
d
= ln 1 +

r0

d
20 V
=1+
exp

r0

20 V
1 = 2.74 cm
d = r0 exp

Again, we used V = 175 V.

23-47 In a certain region, the electric potential is


V (x, y, z) = Axy Bx2 +Cy, where A, B, and C are positive
constants. Find Ex , Ey , and Ez . Where is the electric field
zero (multiple choice)?
V
= Ay + 2Bx
x
V
Ey =
= Ax C
y
V
Ez =
=0
z
Ex =

y = 2BC/A2 .

That analysis gives x and y; any value of z is OK.


23-66 A disk with radius R has a uniform charge density
. (a) By regarding the disk as a series of thin concentric
rings, calculate the electric potential V at a point on the
disks axis a distance x from the center of the disk. Assume
that the potential is zero at infinity. (Hint: Use the result
that the potential at a point on the ring axis at a distance x
from the center of the ring is
V =

Q
1

40 x2 + a2

where Q is the charge of the ring.) (b) Find Ex = V /x.


(a) Using the hint, we can modify the result for the finite ring
to determine the potential dV due to a thin ring of radius r
and charge dQ. We have

along a path radially outward, beginning at the surface of the


cylinder. This will make E and dl parallel, which simplifies
the dot product in the integral. Lets call the radius of the
cylinder r0 and call the distance away from the cylinders
surface that we are looking for d.
Z r=r0 +d
Z f
1
dr
Edl =
V =
2
0 r
r=r0
i
Z r=r0 +d

dr

=
=
[ln (r0 + d) ln r0 ]
20 r=r0
r
20

ln 1 +
=
20
r0

dV =

1
dQ

.
40 x2 + r2

Because electric potential is a scalar quantity, we can integrate dV without keeping track of vectors. First, we must
determine dQ in terms of known quantities. It is equal to
the surface charge density times the surface area dA of the
thin ring,
dQ = dA = 2rdr,
where dA is the product of the length (circumference) 2r
and width dr of the ring of radius r.
Now it is possible to do the integration
Z
Z R
Z R
1
2r dr
r dr

V = dV =
=
2
2
40 0
2
x +r
x2 + r2
0 0
The integral is on the equation sheet:
Z
p
udu

= a2 + u2 .
2
2
a +u
Evaluating it at the limits results in
i
hp 2
x + R2 x .
V =
20
(b) The x component of the electric field is equal to V /x:

1
V
1 2
2 2

=
2x 1
x +R
x
20 2

x
.
=
1
2
20
x + R2
Notice that as R , this result reduces to the field of an
infinite sheet.

Physics 21
Fall, 2011

Solution to HW-7

24-25 A 6.60 F, parallel-plate, air capacitor has a plate


separation of 3.00 mm and is charged to a potential difference
of 300 V. Calculate the energy density in the region between
the plates.
Energy density is calculated using the formula u = 21 0 E 2 .
In this problem, we know the voltage on the capacitor, not
the electric field strength between the plates. But the voltage and electric field strength are related. Because the electric field between the plates is uniform and directed straight
across the gap between the plates, it is easy to show that
V = Ed where d is the separation between the plates. Doing the substitutions and evaluating for the given values, we
get
2

2
V
300 V
12 C
1
1
= 2 8.85 10
u = 2 0
d
N m2
.003 m
= 4.43 10

(a) The reciprocal of the equivalent capacitance for capacitors in series is the sum of the reciprocals of each capacitor.
The equivalent capacitance for capacitors in parallel is the
sum of each capacitor. Using these rules we can break down
the circuit by combining each capacitor until we are left with
a single equivalent capacitance. C3 and C4 are in series so
we can find the equivalent capacitance:
1
1
C3 C4
1
=
+
= C3,4 =
.
C3,4
C3
C4
C3 + C4
Substituting, we find C3,4 = 2.4 F. Next we combine this
with C2 . The capacitors are in parallel so we find:
C2,3,4 = C2 + C3,4 = 7.2 F.
Finally, we combine C1 , C2,3,4 , and C5 which are in series.
1
1
1
1
=
+
+
.
Ceq
C1
C2,3,4
C5
We find Ceq = 2.63 F.

J/m .

24-38 A parallel-plate capacitor has capacitance C0 =


5.00 pF when there is air between the plates. The separation
between the plates is 1.50 mm. (a) what is the maximum
magnitude of charge Q that can be placed on each plate if
the electric field in the region between the plates is not to
exceed 3.00 104 V/m? (b) A dielectric with K = 2.70
is inserted between the plates of the capacitor, completely
filling the volume between the plates. Now what is the maximum magnitude of charge on each plate if the electric field
between the plates is not to exceed 3.00 104 V/m?
(a) Without the dielectric, Q = C0 V and V = Ed. Therefore, the charge for the given values of C0 , E, and d are

4 V
12
Q = C0 Ed = 5.0010
F 3.0010
(.0015 m)
m
= 2.25 1010 C = 225 pC

(b) With the dielectric, C increases to KC0 , and V = Ed


still holds. The charge is K times the previous value:

Q = KC0 Ed = 2.7 2.25 1010 C


= 6.08 1010 C = 608 pC

24-59 In the figure, C1 = C5 = 8.3 F and C2 = C3 =


C4 = 4.8 F. The applied potential is Vab = 230 V. (a)
What is the equivalent capacitance of the network between
points a and b? (b) Calculate the charge on each capacitor
and the potential difference across each capacitor.

(b) We know that for capacitors in series, the charge on


the equivalent capacitor is the same as the charge on each
individual capacitor. Also, for capacitors in parallel, the potential difference across the equivalent capacitor is the same
as the potential difference across each individual capacitor.
Keeping these rules in mind, we will determine the charge
and potential difference across each capacitor by rebuilding
circuit in the opposite way that we broke it down in part
(a), calculating the charge and potential at each point. We
can determine the total charge on Ceq since we know the
potential across a and b.
Qtot = Ceq Vab = 6.1 104 C = 610 C.
Ceq is made up of C1 , C2,3,4 , and C5 , which are in series, so
this charge is the charge on each of these three capacitors,
so we know Q1 = Q5 = 610 C. We can find the potential
difference by applying V = Q/C to find V1 = V5 = 73 V.

We can determine the potential across C2,3,4 the same way


to find V2,3,4 = 84 V. Again, C2,3,4 is made up of C2 and
C3,4 , which are in parallel, so this is the potential difference
across both capacitors. So we know V2 = V3,4 = 84 V, and
applying Q = CV gives Q2 = 400 C and Q3,4 = 200 C.

C3,4 is made up of C3 and C4 in series so we know Q3,4 =


Q3 = Q4 = 200 C. By applying V = Q/C we find V3 =
V4 = 42 V.
Capacitor
1
2
3
4
5

Charge
610 C
400 C
200 C
200 C
610 C

Potential Difference
73 V
84 V
42 V
42 V
73 V

September 21, 2011

24-61
Three capacitors having capacitances of
8.0 F, 8.3 F, and 4.1 F are connected in series across a
40 V potential difference. (a) What is the charge on the
4.1 F capacitor? (b) What is the total energy stored in
all three capacitors? (c) The capacitors are disconnected
from the potential difference without allowing them to
discharge. They are then reconnected in parallel with each
other, with the positively charged plates connected together.
What is the voltage across each capacitor in the parallel
combination? (d) What is the total energy now stored in
the capacitors?

The parallel combination is equivalent to a single effective


capacitance Ceff = C1 + C2 + C2 with the same total charge
3Q. The voltage V = Va Vb is then given by

3 8.2 105 C
3Q
Qtot
=
=
= 12 V
V =
Ceff
C1 + C2 + C3
(8.0 + 8.3 + 4.1) F
One could determine the charges on each capacitor with this
result, using Qi = Ci V .
(d) As in part (b), the total energy stored in all the capacitors can be found by adding up the energies on each capacitor. This time use the form of the equation that involves
the voltage and the capacitance:
Ucap = 21 C1 V 2 + 12 C2 V 2 + 21 C3 V 2 =

1
2

(C1 +C2 +C3 ) V 2

= 21 Ceff V 2 = 21 (2.04 105 F)(12 V)2 = 1.5 mJ.

(a) In a series connection the charges on the plates is as


shown. We can determine the effective capacitance of the
three capacitors and use this to determine the charge Q.
The effective capacitance is found using
1
1
1
1
1
1
1
=
+
+
=
+
+
Ceff
C1
C2
C3
8.0 F 8.3 F 4.1 F
Ceff = 2.04 F
Then

24-65 A parallel-plate capacitor with only air between the


plates is charged by connecting it to a battery. The capacitor is then disconnected from the battery, without any of
the charge leaving the plates. (a) A voltmeter reads 41.0 V
when placed across the capacitor. When a dielectric is inserted between the plates, completely filling the space, the
voltmeter reads 11.5 V. What is the dielectric constant of
this material? (b) What will the voltmeter read if the dielectric is now pulled partway out so it fills only one third of
the space between the plates?
(a) With no dielectric, Q = C0 V0 , where V0 has been measured. With the dielectric, the charge does not change, but
a new V is measured, so Q = KC0 V . Equating both expressions for Q,
C0 V0 = KC0 V

Q = Ceff V = (2.04 F) (40 V) = 8.2 105 C.

K=

V0
41
=
= 3.57 V
V
11.5

(b) This situation is essentialy two capacitors in parallel:

(b) The total energy stored in all the capacitors can be found
by adding up the energies on each capacitor. Use the form of
the equation that involves the charge and the capacitance:

1
Q2
Q2
Q2
1
1
Ucap = 12
+ 12
+ 21
= 21 Q2
+
+
C1
C2
C3
C1
C2
C3

2
5
8.2 10 C
Q2
= 12
= 12
= 1.6 mJ
Ceff
2.04 106 F
Note one gets the same answer using the form Ucap = 21 CV 2 ,
with Ceff and V = 40 V.
(c) When the capacitors are disconnected from the serial circuit, the charge on each one remains the same (+Q). When
they are reconnected in parallel, the total charge (3Q) is free
to move around on the three positive plates shown in the diagram. After the charges equilibrate, the potential difference
Va Vb must be the same across every one of the capacitors.

(2/3) C0

(1/3) KC0

Ceff = 23 C0 + 31 KC0
Therefore the measured voltage will be
V =

Q
=
Ceff

2
3 C0

C0 V0
=
+ 31 KC0

2
3

V0
3V0
= 22.1 V
=
1
K +2
+ 3K

Physics 21
Fall, 2011

Solution to HW-8

25-44 If a 75 W bulb (75 W are dissipated when connected across 120V) is connected across a 220 V potential
difference (as is used in Europe), how much power does it
dissipate?
(a) The power dissipation P and potential difference V
across the bulb are variables, but the resistance R across the
light bulb is a physical constant whether you are in America
or Europe. From the equation sheet we know that P = IV
and V = IR. We can combine these formulas to eliminate
the current I, which we dont know, and get a relation between power, voltage, and resistance:
P = IV =

V2
V2
V
V =
R=
.
R
R
P

Using the US values P = 75 W and V = 120 V, we find


R=

(120 V)2
= 192 .
75 W

We can then use this resistance to solve for the power dissipated when the bulb is connected to the new potential
difference:
P =

V2
(220 V)2
=
= 252 W.
R
192

25-46 A battery-powered global positioning system (GPS)


receiver operating on a voltage of 9.1 V draws a current
of 0.20 A. a) How much electrical energy does it consume
during a time of 2.0 h?

(a) We can model the power supply as a battery with V =


14 kV in series with an internal resistor of Rint . When the
person holds the terminals of the power supply (dont do
this at home, kids), he completes the circuit shown in the
diagram. The total resistance of resistors in series is the sum
Rtot = Rint + Rperson ,
and the current going through them will be the same as the
current going through the equivalent Rtot . This current is
I=

V
14 kV
V
=
=
= 1.167 A.
Rtot
Rint + Rperson
2 k + 10 k

(b) The power dissipated in a device with current running


through it is given by:
P = IV.
We already found the current going through the person in
part (a). However the potential V here is not the same
potential as the battery. It is the potential across just the
persons body, which we can find using V = IRperson :
2

P = IV = I 2 Rperson = (1.167 A) 10 k = 13.6 kW.


(c) To make the battery safe we need to increase the internal
safe
resistance to Rint
. From the second equation in part (a) we
can see that
V
.
Isafe = safe
Rint + Rperson
safe
Solving this equation for Rint
, we have
safe
Rint
=

14 kV
V
Rperson =
10 k = 13.99 M.
0.001 A
0.001 A

(a) We can calculate the energy use over a period of time


if we know the rate that the device uses electrical energy
(power). The power can be determined from:
P = IV = 9.1 V 0.2 A = 1.82 J/s = 1.82 W
The total energy used over a time of two hours is:
Etot = P t = 1.82 J/s 7200 s = 1.31 104 J
25-70 A person with body resistance between his hands
of 10 k accidentally grasps the terminals of a 14 kV power
supply. (a) If the internal resistance of the power supply
is 2000 , what is the current through the persons body?
(b) What is the power dissipated in his body? (c) If the
power supply is to be made safe by increasing its internal
resistance, what should the internal resistance be for the
maximum current in the above situation to be 1.00 mA or
less?

Rperson
Rint

Power supply

September 23, 2011

26-8 Three resistors having resistances of R1 = 1.60 ,


R2 = 2.90 , and R3 = 4.60 are connected in parallel to
a 26.0 V battery that has negligible internal resistance. (a)
Find the equivalent resistance of the combination. (b,c,d)
Find the current in through each resistor. (e) Find the total
current through the battery. (f,g,h) Find the voltage across
each resistor. (i,j,k) Find the power dissipated through each
resistor. (l) Which resistor dissipates the most power: the
one with the greatest resistance or the least resistance?
(a) The equivalent resistance for resistors in parallel is given
by the sum of their reciprocals,
1
1
1
1
=
+
+
.
Req
R1
R2
R3

(a) Write the loop and node equations needed to determine


the currents I1 , I2 , and I3 in the circuit shown. Indicate
clearly the loop used to determine each loop equation.
node:
loop 1:
loop 2:

I2 = I1 + I3
9 I1 2I1 4I2 = 0
8 2I3 1 4I2 = 0

(b) Determine the currents by explicit solution of the equations. You must show your work.
Rearrange loop 1 and loop 2 equations:
loop 1:
loop 2:

3I1 + 4I2 = 9
4I2 + 2I3 = 7

Use the node equation to eliminate I3 = I2 I1 :

Thus,
Req = 0.842 .
(b,c,d) The voltage is the same across parallel resistors,
therefore V = IR I = V /R. Thus,
V
I1 =
= 16.3 A;
R1

V
I2 =
= 8.97 A;
R2

V
I3 =
= 5.65 A.
R3

(e) The current from the battery is the sum of the currents
through each resistor:
Itotal = I1 + I2 + I3 = 30.9 A
One could also use the equivalent resistance,
Itotal = V /Req = 26 V/(0.842 ) = 30.9 A.
(f,g,h) The voltage is the same across resistors in parallel.
V1 = V2 = V3 = 26.0 V
(i,j,k) Any formulation of P = V I = I 2 R = V 2 /R will work,
P1 = 423 W,

P2 = 233 W,

P3 = 147 W.

(l) P = V 2 /R. Since the voltage is the same across all the
resistors, the power is greatest in the smallest resistor.
OE 41-1 For this problem, you must write and then solve
the loop and node equations needed to find the currents I1 ,
I2 , and I3 shown in the figure. (a) In the circuit shown in the
figure, what is the value of the current I1 ? Remember that
I1 may be positive or negative. (b) What is the value of the
current I2 ? Remember that I2 may be positive or negative.
(c) What is the value of the current I3 ? Remember that I3
may be positive or negative.
I1

I3

9V

8V
I2

1V

3I1 + 4I2 = 9
2I1 + 6I2 = 7

(1)
(2)

Multiply (1) by 2 and (2) by 3; add and solve for I2 . Substitute back for I1 then I3 . Results are
I1 = 1.0 A,

I2 = 1.5 A,

I3 = 0.5 A

Physics 21
Fall, 2011

Solution to HW-9

Set up the loop equation for this new loop:


25 V (50 )I = 0 = I =

26-27 In the circuit shown in the figure the batteries have


negligible internal resistance and the meters are both idealized. With the switch S open, the voltmeter reads 15 V. (a)
Find the emf E of the battery. (b) What will the ammeter
read when the switch is closed?

(a) For this part we can ignore the battery with the switch,
since the switch is open and there will be no current through
it. So lets draw the circuit that we are concerned with,
showing also the currents in each branch and the loops we
will use to write Kirchhoffs equations:

25 V
= 0.50 A
50

26-28 In the circuit shown in the figure both batteries have


insignificant internal resistance and the idealized ammeter
reads 1.60 A in the direction shown. (a) Find the emf E of
the battery. (b) Is the polarity shown correct?

(a) To solve this problem, we will use Kirchhoffs rules, but


this time instead of solving for three currents, we will solve
for two currents and the emf. We assign currents I1 and I2 to
the remaining branches and loops as shown in the diagram:

We can find I3 since we were given the measured voltage


across the 50 resistor. Using Ohms Law,
V = IR = I3 =

V
15 V
=
= 0.30 A.
R
50

Now, lets write out the loop and node equations:


Node :
Left loop :

I1 = I2 + I3
E (20 )I1 (75 )I2 = 0

Right loop : E (20 )I1 (30 )I3 (50 )I3 = 0


Since we already know I3 , there are three unknowns in this
system, E, I1 , and I2 . Here are the equations after substituting for I3 and combining some terms:
Node :

I1 = I2 + 0.3

The left loop equation is


0 = 48.0 (I2 ) + 12 (1.6 A) 75.0 V
= 48.0 (I2 ) 55.8 V,
and the right loop equation is
0 = 15.0 (I1 ) + E 48.0 (I2 )
We can solve the left loop equation directly for I2 :
55.8 V
= 1.1625 A.
48.0

Now substitute this I2 into the node equation to solve for I1 :

Now we have three equations for three unknowns. Next we


simplify the equations and solve for E. The solution is
I1 = 0.62 A,

I2 = I1 + 1.60 A.

I2 =

Left loop :
E 20I1 75I2 = 0
Right loop : E 20I1 24 = 0

E = 36.4 V,

The node equation is

I2 = 0.32 A.

(b) The circuit is different for this part and is shown below.
Conveniently, to find the new current in the ammeter, we
only need to consider the one loop shown.

I1 = I2 1.60 A = .4375 A
Substituting the currents found above into the right loop
equations gives
0 = 15.0 (.4375 A) + E 48.0 (1.1625 A)
E = 15.0 (.4375 A) + 48.0 (1.1625 A) = 49.24 V.
(b) The polarity of the battery as shown is correct, because
the E we calculated was positive. If the calculated E had
been negative, it would imply that the assumed polarity in
the drawing was incorrect.

September 30, 2011

26-40 A 12.8F capacitor is connected through a 0.890


M resistor to a constant potential difference of 60.0 V. (a)
Compute the charge on the capacitor at the following times
after the connections are made: 0 s, 5.0 s, 10.0 s, 20.0 s,
and 100.0 s. (b) Compute the charging currents at the same
instants.
(a) As derived, the formula for the charge on a charging
capacitor as a function of time is:

q(t) = Qf 1 et/RC ,

26-48 In the circuit shown below, C = 5.90 F, E = 28.0 V,


and the emf has negligible resistance. Initially the capacitor
is uncharged and the switch S is in position 1. The switch is
then moved to position 2, so the capacitor begins to charge.
(a) What will be the charge on the capacitor a long time
after the switch is moved to position 2? (b) After the switch
has been moved to position 2 for 3.00 ms, the charge on the
capacitor is measured to be 110 C. What is the value of
the resistance R? (c) How long after the switch is moved to
position 2 will the charge on the capacitor be equal to 99.0%
of the final value found in part (a)?

where the final charge Qf = CV = 7.68 104 C. The time


constant = RC = 11.392 s. The table below gives q(t) at
the times specified.
(b) The relationship between charge and current is i = dq/dt,
so we can determine the current as a function of time by
differentiating the expression for q(t) above:
i(t) =

dq
Qf t/RC
=
e
= I0 et/RC
dt
RC

where we substituted Qf = CV and noted that the initial


current I0 that flows is the battery voltage V divided by the
resistance R. I0 is 6.74 105 A.
Here is a table of the charge and current at various times.
Note that 100 s is about nine times the time constant, and
at that point the capacitor is essentially fully charged, and
the current from the battery is essentially zero.
t (s)
0
5
10
20
100

q(t) (C)
0
2.73 104
4.49 104
6.35 104
7.68 104

i(t) (A)
6.74 105
4.35 105
2.80 105
1.16 105
1.00 108

26-43 An emf source with a magnitude of E = 120 V, a


resistor with a resistance of R = 87.0 , and a capacitor
with a capacitance of C = 3.90 F are connected in series.
As the capacitor charges, when the current in the resistor is
0.700 A, what is the magnitude of the charge on each plate
of the capacitor?
(a) The simplest way is to apply the loop equation. Let q(t)
and i(t) be the instantaneous charge on the capacitor and
current in the circuit. Then
E VR VC = 0
q(t)
E i(t)R
= 0.
C
Now we can solve for the charge on the capacitor as a function of the current:
h
i
q(t) = C[E i(t)R] = 3.90 C 120 V (0.7 A)(87 )
= 230 C.

(a) We know that after a long time the circuit will approach
a steady state where the charge on the capacitor will be
simply given by
Q = CE.
Substituting C = 5.90 F and E = 28.0 V we find the charge
on the capacitor after a long time will be Q = 165.2 C.
(b) We know that the resistance R is part of the time constant in the function q(t). We found q(t) by solving the
differential equation obtained from Kirchoffs loop equation.
t

q(t) = CE(1 e RC )
Since we know q(t = 3 s) we can rearrange this equation to
solve for R.
1
t
R=
q
C ln(1 CE
)
We insert our value for q(t = 3 s) = 110 C and find the
resistance R = 464 .
(c) We want to find at what time t will the charge on the
capacitor be 99% of its final value which we found in part (a).
In other words we want to solve for t when q = 0.99 CE.
We rearrange our equation for q(t) to get:
t = RC ln(1

q
).
CE

Substituting our values of R, C, and q we find:


t = (464 )(5.9 106 F) ln(1 0.99) = 0.0126 s

Physics 21
Fall, 2011

Solution to HW-10

27-3 In a 1.35 T magnetic field directed vertically upward,


a particle having a charge of magnitude 8.90 C and initially
moving northward at 4.72 km/s is deflected toward the east.
(a) What is the sign of the charge of this particle? (b) Find
the magnetic force on the particle.

The magnetic force on a charged particle moving in a magnetic field is given by the equation
F = qv B
Since, in this case, F, v, and B are mutually perpendicular,
the magnitude of F is simply given by F = qvB, with the
direction determined by the right hand rule.
(a) We need to apply the right hand rule to see if the direction of the force is consistent with a positive charge or a
negative charge. Imagine you are seated so that north is in
front of you. The other directions are then determined: east
is to the right, south is behind you, and west is to the left.
So northward velocity means the particle is moving forward.
Point the fingers of your right hand straight forward. The
magnetic field is upward, so curl the fingers of your right
hand upward. In order to do this, your palm must be facing
upward. Then, the thumb of your right hand is pointing to
the right (eastward). Eastward is the direction the particle
is deflected. Thus, the particle must have a positive charge.
Here is a diagram:
N
v

27-4 A particle with mass m = 1.81103 kg and a charge


of q = 1.22 108 C has, at a given instant, a velocity
v = (3.00 104 m/s) j. (a) What is the magnitude of the
particles acceleration produced by a uniform magnetic field
B = (1.63 T) i + (0.980 T) j? (b) What is the direction of
the particles acceleration?
To determine the acceleration of the particle we need to know
what force is acting on it. We can assume the only force is
due to the magnetic field. The force on a charged particle
moving in a magnetic field is given by:
F = q (v B)
For the cross product, we notice that many components of
v and B are zero:
v = vyj and B = Bxi + Byj.
We can evaluate the cross product v B by using the cross
product of each pair of unit vectors:
j i = k

j j = 0

Then
i

F = qv B = q vyj Bxi + Byj = qvy Bx k


Substituting the given quantities we get:
F = (1.22 108 C)(3.00 104 m/s)(1.63 T)

= 5.97 104 N k
Using Newtons second law, we can find the acceleration of
the particle:

a = F/m = 0.330 m/s k.

(b) F = qvB = (8.90 C)(4.72 km/s)(1.35 T) = 0.0567 N

September 30, 2011

27-21 A deuteron (the nucleus of an isotope of hydrogen)


has a mass of 3.34 1027 kg and a charge of 1.60 1019 C.
The deuteron travels in a circular path with a radius of
6.90 mm in a magnetic field with a magnitude of 2.60 T. (a)
Find the speed of the deuteron. (b) Find the time required
for it to make 21 of a revolution. (c) Through what potential difference would the deuteron have to be accelerated to
acquire this speed?

Note that Mastering Physics is only asking us to provide |V |,


so we dont have to worry about getting the signs right.
K0 + U0 = K1 + U1
K1 K0 = U0 U1
2
1
2 mv = q (V0 V1 ) = qV

2
3.34 1027 kg 8.594 105 m/s
mv
V =
=
2q
2 (1.60 1019 C)
|V | = 7709 V = 7.71 kV
27-25 An electron in the beam of a TV picture tube is accelerated by a potential difference of 1.95 kV. Then it passes
through a region of transverse magnetic field, where it moves
in a circular arc with a radius of 0.179 m. What is the magnitude of the field?
From problem 27-24, we know that B = mv/(eR), but we
have to find v for an electron accelerated through a potential
difference of V = 1950 volts. By definition, the electron
gains an energy 1950 eV. Assuming the electron starts from
rest, we have
2
1
2 mv

(a) As is seen in the figure, the force on a moving charged


particle due to a magnetic field causes it to travel in a circular
path. We have an equation from the equation sheet that
describes this motion.
R=

mv
qB

Here, the speed is written as v to remind us that only


the component of the velocity vector that is perpendicular
to the magnetic field direction contributes to the circular
motion. In this problem, we are told it is circular motion,
so we know that the velocity is purely perpendicular to the
magnetic field direction. We are given numbers for all of the
other quantities in this equation, so simply solve it for v .

6.90 103 m 1.60 1019 C (2.60 T)


RqB
=
v =
m
3.34 1027 kg
= 8.594 105 m/s
(b) For this part, we must recall concepts from Physics 11.
We are looking for the time t to complete half a revolution, or
to travel a distance d of half the circumference of the circular
orbit. Since d = R, and the speed v of the particle in its
circular orbit is constant, we have
R = v t = t = R/v
Then

6.90 10 m
R
=
= 2.52 108 s = 25.2 ns
t=
v
8.594 105 m/s
(c) This type of problem can be solved by conservation of
energy. See example 23.7 in the textbook for a reminder.
The particles gain in kinetic energy is equal to its loss in
potential energy. In this case, the particle starts from rest.

= eV = (1950 eV) 1.602 1019 J/eV

Solving for v gives v = 2.62 107 m/s. Then


B=

mv
= 8.32 104 m.
eR

OH11-05 An electron enters a uniform magnetic field with


magnitude 0.3 T at a 45 angle to B. Determine the radius r
and pitch p (distance between loops) of the electrons helical
path, assuming its speed is 2 106 m/s.

Since v makes an angle of 45 with B, the components of v


parallel and perpendicular to B (vk and v ) both have the

value v0 / 2 (since sin = cos = 1/ 2 for = 45 ), where


v0 is the given speed 2.0 106 m/s. We get the radius of the
circular motion using v :

(9.11 1031 )(2.0 106 / 2)


me v
=
= 26.8 m
r=
eB
(1.6 1019 )(0.30)
The time to make one loop of circular motion is
t =

2r
2me v
2me
=
=
.
v
eBv
eB

The pitch is the distance travelled parallel to B (at speed


vk ) in time t (one loop):
p = vk t =

2me vk
= 1.69 104 m = 169 m
eB

Physics 21
Fall, 2011

Solution to HW-12

27-36 A straight, vertical wire carries a current of I =


1.19 A downward in a region between the poles of a large
superconducting electromagnet, where the magnetic eld B
has a magnitude of B = 0.591 T and is horizontal. What
are the magnitude and direction of the magnetic force on a
section of the wire with a length of l = 1.00 cm that is in
this uniform magnetic eld, if the magnetic eld direction is
(a) east, (b) south, (c) 34 south of west?

27-39 A thin, 51.0 cm long metal bar with mass 770 g


rests on, but is not attached to, two metallic supports in
a uniform magnetic eld with a magnitude of 0.470 T, as
shown in the gure. A battery and a resistor of resistance
26.0 are connected in series to the supports. (a) What
is the largest voltage the battery can have without breaking the circuit at the supports? (b) The battery voltage has
the maximum value calculated in part a. If the resistor suddenly gets partially short-circuited, decreasing its resistance
to 2.70 , nd the initial acceleration of the bar.

The diagram shows the view looking down in the direction


of the current, for an arbitrary orientation of B in the horizontal plane:

N
B
W

E
Fmag

We use the right hand rule to evaluate the magnetic force


Fmag = Il B,
where l is a vector of length l, the length of the wire section,
that points in the direction of the current. One can see
that for any horizontal B, the magnetic force will also be
horizontal, and its direction will be 90 clockwise from B.
(a) For B pointing east, the magnitude of Fmag is
Fmag = IlB = (1.19 A)(0.010 m)(0.591 T) = 7.03 mN
and the direction is south.
(b) For B pointing south, Fmag is the same as in part (a),
and the direction of Fmag is west.
(c) For B pointing east, Fmag is the same as in part (a), and
the direction of Fmag is 34 west of north.

The voltage V causes a current I = V /R to ow in the


wire. Then the magnetic eld exerts a force on the currentcarrying wire that is in the upward direction.
(a) The largest force that wont break the circuit occurs when
the upward magnetic force just balances the downward gravitational force:


V


Fmag = IL B = ILB = LB = mg,
R
where I is the current through the wire, and L is a vector
that points along the portion of the wire that is in the eld.
Solving for V , we nd
V =

mgR
= 819 V.
LB

(b) When the resistance R drops to a lower value R , the


current in the wire increases, and the upward force on the
wire will exceed the downward gravitational force, leading
to a net upward force ma:
V LB
mg = ma.
R
Substituting the expression for V derived in part (a) into the
expression above leads to


mgR LB
R
mg = ma g  g = a
LB
R
R


R R
2
a=g
= 84.6 m/s .
R

October 10, 2011

27-42 The plane of a rectangular loop of wire with a width


of 5.0 cm and a height of 8.0 cm is parallel to a magnetic eld
of magnitude 0.16 T. The loop carries a current of 6.3 A. (a)
What torque acts on the loop? (b) What is the magnetic
moment of the loop? (c) What is the maximum torque that
can be obtained with the same total length of wire carrying
the same current in this magnetic eld?

27-74 A wire 28.0 cm long lies along the z axis and carries a current of 8.60 A in the +z direction. The magnetic eld is uniform and has components Bx = 0.200 T,
By = 0.968 T, and Bz = 0.327 T. Find the x, y, and
z components of the magnetic force on the wire, and the
magnitude of that force.
The force on the wire is given by

(b) The magnetic moment of the loop has magnitude IA


and direction perpendicular to the loop. The magnitude is
= 2.52 102 A m2
(a) The torque on the loop is = B. Because is
perpendicular to B, the magnitude of is
= B = 4.03 103 N m
(c) The maximum torque would be achieved for the loop
with the largest area for the available length of wire. The
shape of the optimum loop is a circle. If the sides are a and
b, then the radius r of the circle satises
2r = 2(a + b)

r=

a+b
= 0.13 m.

The magnitude of the maximum torque is


max = r2 IB = 5.42 103 N m.
27-43 In the Bohr model of the hydrogen atom, in the
lowest energy state the electron orbits the proton at a speed
of v = 2.2 106 m/s in a circular orbit of radius r = 5.3
1011 m. (a) What is the orbital period of the electron? (b)
If the orbiting electron is considered to be a current loop,
what is the current I? (c) What is the magnetic moment of
the atom due to the motion of the electron?
(a) The time T to travel distance 2r at speed v is
T = 2r/v =

2(5.3 1011 m)
= 1.5 1016 s
2.2 106 m/s

(b) Current is the amount of charge Q passing a point in


time t. Since the electron with charge e passes any point
on its orbit once in time T ,
I = e/T =

1.602 1019 C
= 1.1 mA
1.5 1016 s

(c) The magnitude of the magnetic moment of a current loop


is the current I times the area of the loop:
= r2 I = (5.3 1011 m)2 (.00106 A)
= 9.3 1024 A m2

(Bxi + Byj + Bz k).

F = IL B = ILk
i = j,
We can work out the cross products using k

k j = i, and k k = 0. We get


F = IL Byi + Bxj Fx = 2.33 T, Fy = 0.482 T

F = Fx2 + Fy2 + 02 = 2.38 T

Physics 21
Fall, 2011

Solution to HW-13

28-12 Two parallel wires are 5.00 cm apart and carry currents in opposite directions, as shown in the figure. Find
the magnitude and direction of the magnetic field at point
P due to two 1.50-mm segments of wire that are opposite
each other and each 8.00 cm from P .

28-18 Two long, straight wires, one above the other, are
separated by a distance 2a and are parallel to the x axis. Let
the +y axis be in the plane of the wires in the direction from
the lower wire to the upper wire. Each wire carries current
I in the +x direction. Find B, the net magnetic field of the
two wires at the following points in the plane of the wires:
(a) midway between the wires, (b) at a distance a above the
upper wire, and (c) at a distance a below the lower wire.

(b)
I

dl

2a

(a)
I

r-r'
dl

r-r'

(c)

The magnitude of the magnetic field near a long wire is


B=

Use the Biot-Savart Law:


dB =

0 Idl (r r )
,
3
4
|r r |

where dl points in the direction of the current I, and r r


points from the current point to the field point (P ).
We can evaluate dBtop and dBbottom (the contributions to
the total field from the top and bottom wires, respectively)
separately and then add them. The diagram shows the vectors dl and rr for each of these contributions. The simplest
way to evaluate the cross product is to use the right hand
rule for the direction, and to get the magnitude from the
products of the magnitudes |dl| and |r r | and the sin of
the angle between the vectors.
By the right hand rule, both dBtop and dBbottom point into
the page. The magnitudes of dl and r r are 0.0015 m and
0.08 m, respectively, in each case. Since the wires are 5.0 cm
apart, the sines of the angles in each case is 2.5/8.0. Hence
(12.0)(0.0015)(0.080)(2.5/8.0)
107
0.083

dBtop

dBbottom

= 8.79 108 T
(24.0)(0.0015)(0.080)(2.5/8.0)
= 107
0.083
7
= 1.76 10 T.

Since both contributions to the field point into the page, so


does the sum, and
dBtotoal = dBtop + dBbottom = 2.64 107 T.

0 I
,
2R

where R is the perpendicular distance to the wire. The direction of the field is given by the right hand rule. For the
present problem, we must add the vector contributions to
the field from each wire at each point (a), (b), and (c). The
distances will be multiples of a.
(a) At point (a), midway between the wires, the field from
the top wire is 0 I/(2a) into the page, and the field from
the bottom wire is 0 I/(2a) out of the page. The vector
sum is zero.
B=0
(b) At point (b), a distance a above the top wire, the field
from the top wire is 0 I/(2a) out of the page, and the field
from the bottom wire is 0 I/(23a) out of the page. The
sum is

20 I
1
0 I 1
=
+
(out of the page).
B=
2 a 3a
3a
In terms of vector components,
B=

20 I
k.
3a

(c) At point (c), a distance a below the bottom wire, the


field from the bottom wire is 0 I/(2a) into the page, and
the field from the top wire is 0 I/(23a) into the page.
B=

20 I
3a

(into the page).

In terms of vector components,


B=

20 I
k.
3a

October 12, 2011

Physics 21
Fall, 2011

Solution to HW-14

Cancelling Magnetic Field Four very long, currentcarrying wires in the same plane intersect to form a square
with side lengths of 39.0 cm, as shown in the figure. The currents running through the wires are 8.0 A, 20.0 A, 10.0 A,
and I. Find the magnitude and direction of the current I
that will make the magnetic field at the center of the square
equal to zero.

vector sum of the fields can only be zero on the line between
the wires.
The magnitude of the field B a distance R from a long wire
with current I is B = 0 I/(2R). The vector field will be
zero at a point on the line between the wires a distance x
from the left wire and L x from the right wire, where the
magnitudes of the fields are equal. Then
0 I2
I1
I2
0 I1
=
=
=
2x
2(L x)
x
Lx
Solving this equation and substituting values leads to

I1
23
x=
38 cm = 8.7 cm
L=
I1 + I2
23 + 78
MasteringPhysics asks for the distance from the 78 A wire
in the direction of the 23 A wire, which is L x = 29.3 cm
in our notation.
(b) Panel (b) of the diagram is similar to panel (a); it shows
the two wires and the fields when the currents are in opposite
directions. In this case the vector sum of the fields can only
be zero on the line connecting the wires, but outside the
wires. We consider first a distance x to the left of I1 ; in this
case the condition is
0 I2
I1
I2
0 I1
=
=
=
2x
2(L + x)
x
L+x

The field point at the center of the square is equidistant from


all four wires. Let this distance be d = 21 0.39 m. We just
have to keep track of the direction of each field using the
right hand rule. Let out of the page be plus, and let I > 0
correspond to up:
B out of page =

0
(10 + I 8 + 20)
2d

Solving, we get I = 2 A, and the minus sign means I is


directed downward.
28-22 Two long, parallel transmission lines, L = 38.0 cm
apart, carry currents I1 = 23.0-A and I2 = 78.0-A. Find all
locations where the net magnetic field of the two wires is
zero if these currents are (a) in the same direction or (b) in
opposite directions.
(b)

(a)
L
x
I1

x
I2

I1

I2

(a) Panel (a) of the diagram shows the two wires end on
when the currents are in the same direction. L is the distance
between the wires. The magnetic field lines due to each wire
separately are shown by the concentric circles (dashed for I1 ,
solid for I2 ). The direction of the field follows from the right
hand rule and is shown at selected points by an arrow next
to each circle. By looking at the directions of the two fields
in various locations, its easy to see that for case (a), the

Solving this equation and substituting values leads to

I1
23
x=
38 cm = 15.9 cm
L=
I2 I1
78 23
MasteringPhysics asks for the distance from the 78 A wire
in the direction of the 23 A wire, which is L + x = 53.9 cm
in our notation.
What about a point to the right of I2 in panel (b)? We can
set up the equation; a point a distance x to the right of I2
would be x + L from I1 . Then

I1
I2
I2
L
=
= x =
x
L+x
I1 I2
This solution wont work in our case. I2 > I1 , so x < 0,
which contradicts our initial assumption that x is a positive
distance. Therefore there are no other solutions.
Comparing the two solutions we have obtained for part (b),
one can see that a general way of writing the solution is

I<
L,
x=
I> I<
where I< (I> ) is the lesser (greater) of I1 and I2 . The point
where the field is zero is outside the two currents, a distance
x from the wire with the smaller current.

October 14, 2011

Wire and Square Loop A square loop of wire with side


length a carries a current I1 . The center of the loop is located
a distance d from an infinite wire carrying a current I2 . The
infinite wire and loop are in the same plane; two sides of the
square loop are parallel to the wire and two are perpendicular
as shown. (a) What is the magnitude F of the net force on
the loop? (b) The magnetic moment of a current loop
is defined as the vector whose magnitude equals the area of
the loop times the magnitude of the current flowing in it
( = IA), and whose direction is perpendicular to the plane
in which the current flows. Find the magnitude F of the
force on the loop from Part (a) in terms of the magnitude of
its magnetic moment.

(a) The B field is into the page everywhere on the right of


the wire in the plane of the square loop. Its magnitude is
given by B = 0 I2 /2R, where R is the distance to the wire.
The diagram shows the forces on the left and right hand side
of the loop from this field. These forces are obtained from
F = Il B, and the magnitudes are
0 I2
2(d a/2)
0 I2
= I1 aB = I1 a
2(d + a/2)

Fleft = I1 aB = I1 a
Fright

There is also an upward force (Ftop ) and (Ftop ), but they


are equal in magnitude and oppositely directed. Hence the
net force is the vector sum of the forces to the left and to
the right. The net force is to the left and has magnitude

1
1
0 I1 I2 a

Fleft Fright =
2
d a/2 d + a/2
0 I1 I2
a2
=
2
2 d a2 /4
(b) The magnitude of the magnetic moment of the loop is
the current times the area, or
2

= I1 a .
We can write the net force in terms of as
Fleft Fright =

I2
0
2 d2 a2 /4

28-26 Two long, parallel wires are separated by a distance


of d = 2.70 cm. The force per unit length that each wire
exerts on the other is 4.10 105 N/m, and the wires repel
each other. The current in one wire is I1 = 0.700 A. (a)
What is the current in the second wire? (b) Are the two
currents in the same direction or in opposite directions?
I1

B1
I2

Lets assume that the currents flow in the directions shown,


and well show that the force between the wires is repulsive.
The magnitude of the magnetic field B1 of wire 1 at wire 2
is
0 I1
,
B1 =
2d
and from the rh rule, B1 points into the page. The magnitude of the force F that B1 exerts on a length L of wire 2
is
0 I1
F = I2 LB1 = I2 L
,
2d
and by the rh rule one can see that the direction is downward.
One can go through a similar argument to find that the force
on the upper wire has the same magnitude and is upward,
so the forces make the wires repel each other.
The force per unit length is
F
0 I1 I2
=
L
2 d
The question asks for I2 , so

0.027
2 F d
= 0.5107 4.10 105
= 7.91 A
I2 =
0 L I1
0.7
In general, the currents must flow in the opposite direction.
28-27 The wires in a household lamp cord are typically
d = 2.5 mm apart center to center and carry equal currents
in opposite directions. (a) If the cord carries current to a 100
watt light bulb connected across a 120 V potential difference,
what force per meter does each wire of the cord exert on the
other? (Model the lamp cord as a very long straight wire.)
(b) Is the force attractive or repulsive? (c) Is this force large
enough so it should be considered in the design of lamp cord?
Since P = IV , the current in each wire is I = P/V =
100 W/120 V = 0.833 A.
(a) The force per unit length is given by the formula derived
in problem 28-26, with both currents equal:
F
0 I 2
(0.833)2
=
= 2 107
= 5.56 105 N/m
L
2 d
0.0025
(b) The force will be repulsive.
(c) No. The force is small compared to the gravitational
force. If we guess that a meter of wire weighs a few ounces,
say 0.1 kg, then
2

mg = (0.1 kg)9.81 m/s 1 N,


much larger than the magnetic force.

Physics 21
Fall, 2011

Solution to HW-15

28-32 A solid conductor with radius a is supported by insulating disks on the axis of a conducting tube with inner
radius b and outer radius c. The central conductor and tube
carry equal currents I in opposite directions. The currents
are distributed uniformly over the cross sections of each conductor. (a) Derive an expression for the magnitude of the
magnetic field at points outside the central, solid conductor,
but inside the tube (a < r < b). (b) Derive an expression
for the magnitude of the magnetic field at points outside the
tube (r > c).

28-34 A closely wound coil has a radius a = 5.90 cm and


carries a current I = 3.30 A. How many turns must it have
if, at a point on the coil axis 6.20 cm from the center of the
coil, the magnetic field is 6.58 104 T?
The magnitude of the magnetic field at the point x on the
axis of a single circular loop with a radius a and current I is
B=

0 Ia2
.
2(x2 + a2 )3/2

For a loop with N turns, the total magnetic field will be N


times this magnitude. So we can determine the number of
turns by solving for N to get:
2B(x2 + a2 )3/2
0 Ia2
3/2

2(.000658) .0622 + .0592


= 57
=
(4 107 )(3.30)(.059)2

N=

(a) This problem is best answered with Amperes Law


I
B dl = 0 Iencl
Chose a circular line integral path with radius r between the
center and outer conductor, then by symmetry we expect
the value of B to be constant around the path. The only
current enclosed is the current I in the center conductor.
2rBinside = 0 I
Solving for B,
Binside =

I
0 I
= 2 107
2r
r

(b) Taking a similar approach for this problem, choose a


current loop with radius r > c. The total current enclosed is
now zero because the two currents I in each conductor are
going in opposite directions.
Boutside = 0

October 18, 2011

28-36 The figure shows, in cross section, several conductors


that carry currents through the plane of the figure. The
currents have the magnitudes I1 = 4.0 A, I2 = 6.5 A, and
I3 = 2.1 A, and the directions shown. Four paths,H labeled a,
b, c, and d, are shown. What is the line integral B dl for
each of the four paths? The integral involves going around
the path in the counterclockwise direction.

28-41 A solenoid is designed to produce a magnetic field


of 2.00 102 T at its center. It has a radius of 1.60 cm
and a length of 33.0 cm, and the wire can carry a maximum
current of 13.5 A. (a) What minimum number of turns per
unit length must the solenoid have? (b) What total length
of wire is required?
(a) The magnitude of the magnetic field inside a solenoid
is given by B = 0 nI, where n is the number of turns per
unit length. Solving this equation for n = B/0 I, we can
see that the minimum n will occur when the current is at its
maximum;
n=

turns
2.00 102 T
B
= 1180
=
.
Wb
7
0 Imax
m
(4 10 Am )(13.5 A)

(b) The total length of wire L will be the total number of


loops times the length of one loop, which is the circumference, L = N 2r. The total number of loops is simply the
number of turns per length times the total length of the
solenoid N = nl. Thus the total length of the wire is
L = nl2r = (1180 m1 )(.33 m)(2)(.016 m) = 39.1 m

Use Amperes Law,


I

B dl = 0 Iencl ,

where the positive direction of current flow is out of the page


since the integral is done counterclockwise.
(a)
I
Iencl = 0

28-45 A wooden ring whose mean diameter is 16.0 cm is


wound with a closely spaced toroidal winding of 585 turns.
(a) Compute the magnitude of the magnetic field at the center of the cross section of the windings when the current in
the windings is 0.655 A.

B dl = 0

(b)
Iencl = I1

B dl = 5.03 106 T m.

(c)
Iencl = I2 I1

B dl = 3.14 106 T m.

(d)
Iencl = I2 + I3 I1

B dl = 5.78 106 T m.
(a) In the center of the windings, the strength of the magnetic field can be found using the formula derived in the class
lecture:
B = 0 nI
This is the same formula used for a solenoid; a toroid is
basically a rolled up solenoid. n is the number of turns per
unit length along the toroid. To calculate n, we use the
number of turns divided by the circumference of the toroid:
n=
B = 0

585
0.16

585
0.655 = 9.58 104 T
0.16

Physics 21
Fall, 2011

Solution to HW-16

F =

28-48 The current in the windings of a toroidal solenoid


is 2.400 A. There are N = 500 turns and the mean radius is
r = 25.00 cm. The toroidal solenoid is filled with a magnetic
material. The magnetic field inside the windings is found
to be 1.940 T. (a) Calculate the relative permeability. (b)
Calculate the magnetic susceptibility of the material that
fills the toroid.
(a) The magnetic field inside a tightly wound toroidal
solenoid is
0 N I
B = Km 0 nI = Km
,
2r
where n is the number of turns per unit length and N is the
total number of turns.
Solving the last equation for Km , we get
Km =

Using SI units,

2rB
2 0.25 1.94
=
= 2021.
0 N I
(4 107 ) 500 2.4

(b) The magnetic susceptibility is


m = K m 1
thus the answer is 2020.
28-52 A long, straight wire carries a current of 2.50 A. An
electron is traveling in the vicinity of the wire. At the instant
when the electron is 4.50 cm from the wire and traveling with
a speed of 6.00 104 m/s directly toward the wire, what are
the magnitude and direction (relative to the direction of the
current) of the force that the magnetic field of the current
exerts on the electron?

I
The magnetic field due to the wire has magnitude
B=

0 I
2r

and direction (from the rh rule) out of the page at the location of the electron as shown. The force on the electron is
given by
F = qv B = ev B.

(1.602 1019 )(60000)(4 107 )(2.5)


N
2(.045)

= 1.07 1019 N
The magnetic field exerts a force in the same direction as
the current.
28-55 Two identical circular, wire loops 35.0 cm in diameter each carry a current of 2.30 A in the same direction.
These loops are parallel to each other and are 22.0 cm apart.
Line ab is normal to the plane of the loops and passes through
their centers. A proton is fired at 2750 m/s perpendicular
to line ab from a point midway between the centers of the
loops. Find the magnitude of the magnetic force these loops
exert on the proton just after it is fired.
This problem involves the magnetic force on a moving
charged particle, F = qv B. The trick is to find the magnetic field at the position of the particle due to the two loops.
The particle is located on line ab. From the description of
the loops, we know that line ab is on the axis of both loops.
Section 28-5 in the textbook shows how to calculate the magnetic field along the axis of a loop. In particular, for a loop
of radius a, carrying a current I, the magnetic field along the
axis a distance x from the loop is given by equation 28.15:
B=

0 Ia2
2 (x2 + a2 )

3/2

The direction of the magnetic field is along the axis as determined by the right hand rule. Notice that the direction
of the force is the same regardless of which side of the loop
is particle is located at.
Since the particle is located equidistant from the two loops,
and because the loops carry the same current in the same
direction, each loop contributes the same magnitude and direction of magnetic field. The total magnetic field is thus
twice the magnetic field due to a single loop.
We only need to find the magnitude of the force, F =
qvB sin = qvB sin 90 = qvB, since the direction of v is
perpendicular to the loop axis, and the direction of B is
along the loop axis.

!
0 Ia2
0 Ia2
F = qv 2
= qv
3/2
3/2
2 (x2 + a2 )
(x2 + a2 )

= 1.60 1019 C (2750 m/s)

2
4 107 T m/A (2.30 A) (.175 m)

3/2

2
2
(.110 m) + (.175 m)
= 4.42 1021 N

F is in the direction shown in the diagram. Note that the


negative sign of the electron makes F in the opposite direction from v B. The vectors F, v, and B are mutually
perpendicular, so the magnitude of F is
F = evB =

ev0 I
2r

October 21, 2011

28-63 Two long, parallel wires hang by 4.00-cm-long cords


from a common axis (see the figure ). The wires have a mass
per unit length of 1.10 102 kg/m and carry the same
current in opposite directions. (a) What is the current in
each wire if the cords hang at an angle of 6.00 with the
vertical?

(a) Two parallel wires with currents running in opposite directions exert a repulsive force on one another. Using the
formula for the magnetic field at a distance r from a long
wire, B = 0 I/(2r), and the force F on a length L of wire
from a magnetic field B perpendicular to the wire, F = ILB,
one can find the force per unit length on one wire due to the
current in the other:
0 II
F
=
.
L
2r
Viewing a wire end-on allows us to construct a free body
diagram involving the gravitational force, tension, and the
magnetic force.

The sum of the forces can then be written as


X
Fy = T cos mg = 0 = T = mg/ cos
X

Fx = F T sin = 0 = F = T sin = mg tan ,

where F denotes the force on a length of wire L. Now we


use the expression for the magnetic force F and the linear
mass density, = m/L,
0 I 2 L
= mg tan = Lg tan
2r
r
0 I 2 L
1
= Lg tan = I =
2rg tan
2r
0

F =

Note the Ls cancel. The distance r between the two wires is


twice the base leg of the right triangle that is formed when
the 4.00-cm cord hangs at 6 with respect to the vertical.
r = 2 l sin = 2(0.040 m) sin(6.00 ) = 8.362 103 m.
Now we substitute this r and 0 = 4 107 T m/A,
= 1.10 102 kg/m, g = 9.8 m/s2 , and = 6.0 into
the expression for I and find
I = 21.8 A,

When I is increased, the angle from the vertical increases.


A large current is required for even the small displacement
seen here.
29-5 A circular loop of wire with a radius of r = 14.0 cm
and oriented in the horizontal xy plane is located in a region
of uniform magnetic field. A field of magnitude B = 1.8 T is
directed along the positive z direction, which is upward. (a)
If the loop is removed from the field region in a time interval
of 2.0 ms, find the average of the emf that will be induced
in the wire loop during the extraction process. (b) If the
coil is viewed looking down on it from above, is the induced
current in the loop clockwise or counterclockwise?

(a) The induced emf is given by Faradays Law:


d
.
= |E|
E =
dt
t

The change in flux is the difference between the final


flux and the initial flux, and t is 2.0 s.
Since the field B is initially perpendicular to the loop, the
initial flux i through the loop is just the area of the circular
loop times the magnetic field:
Z
i = B dA = r2 B.
After the loop is removed from the magnetic field, the final
flux f is zero. We get the absolute value of the emf from

2
f i 0 i
=
= i = r B
|E| =

t
t
t
t
2
(0.14 m) 1.8 T
=
= 55 V.
0.002 s
(b) The direction of the induced current I is obtained using
Lenzs Law. The direction of I must oppose the change
in flux. Originally, the flux was determined by the field B
pointing up (as one looks down on the loop). When the
loop is removed from the field, the induced current will be
in the direction that will restore an upward pointing field.
Therefore the current is counterclockwise.

Physics 21
Fall, 2011

Solution to HW-17

29-7 The current in the long, straight wire AB shown in


the figure is upward and is increasing steadily at a rate di/dt.
(a,b) At an instant when the current is i, what are the magnitude and direction of the field B at a distance r to the
right of the wire? (c) What is the flux dB through the
narrow shaded strip? (d) What is the total flux through the
loop? (e) What is the induced emf in the loop? (f) Evaluate the numerical value of the induced emf if a = 12.0 cm,
b = 36.0 cm, L = 24.0 cm, and di/dt = 9.60 A/s.

29-8 A flat, circular, steel loop of radius 75 cm is at rest


in a uniform magnetic field, as shown in an edge-on view
in the figure. The field is changing with time, according to
1
B(t) = (1.4 T)e(0.057s )t . (a) Find the emf induced in the
loop as a function of time (assume t is in seconds). (b) When
is the induced emf equal to 1/20 of its initial value? (c) Find
the direction of the current induced in the loop, as viewed
from above the loop.

(a) We can find the emf induced in the loop using


I
Z
d
d
E dl =
B dA = ,
dt
dt
(a,b)
B=

0 i(t)
2r

into page (by right hand rule)

(c)
0 i(t)
L dr
2r

d = B dA =
(d) Integrate
=

d =

0 i(t)L
2

dr
0 i(t)L b
=
ln
r
2
a

(e)
d
0 L di b
|E| =
=
ln
dt
2 dt a
(f)
(2 10

36
)(0.24 m)(9.6) ln
12

= 5.06 107 V

What will be the direction of the current? Counterclockwise.

where the integral of E dl is the induced emf. Because the


magnetic field makes a 60 degree angle with the loop, we can
write
Z
B dA = = B(t)A sin(60)
where A = r2 is the area of the loop. Because the area
of the loop doesnt change with time, d/dt = r2 dB/dt,
where
d
B(t) = (1.4)(0.057)e0.057t
dt
Putting these pieces together results in:
E = r2 sin(60)(1.4)(0.057)e0.057t = 0.122e0.057t
We usually take E to be the magnitude of the emf, and dont
worry about the sign. The negative sign is related to the
polarity of the voltage.
(b) Now that we have an expression for the induced emf as
a function of time, we can solve for t at a particular emf.
1
= e0.057t = t = ln 20/(0.057 s1 ) = 52.6 s
20
(c) According to Lenzs Law, the induced emf will cause
current to flow in the direction that opposes the change in
magnetic flux. Because the magnitude of the magnetic field
directed upwards through the loop is decreasing with time,
the induced emf will cause a current to flow in a direction
that will cause a magnetic field upward. For that to happen,
the current must flow in a counterclockwise direction.

October 27, 2011

29-11 In a region of space, a magnetic field points in the


+x direction (toward the right). Its magnitude varies with
position according to the formula Bx = B0 + bx, where B0
and b are positive constants, for x 0. A flat coil of area A
moves with uniform speed v from right to left with the plane
of its area always perpendicular to this field. (a) What is the
emf induced in this coil while it is to the right of the origin?
(b) As viewed from the origin (which we take to be to the
left), what is the direction (clockwise or counterclockwise) of
the current induced in the coil? (c) If instead the coil moved
from left to right, what would be the answer to part (a)? (d)
If instead the coil moved from left to right, what would be
the answer to part (b)?

29-13 The armature of a small generator consists of a flat,


square coil with 120 turns and sides with a length of 1.95 cm.
The coil rotates in a magnetic field of 7.80 102 T. (a) What
is the angular speed of the coil if the maximum emf produced
is 2.60 102 V?
(a) The figure below shows the general configuration of the
square coil and the magnetic field. Note that our coil has
120 turns. The magnetic field is constant and to the right,
while the coil (and consequently, the vector A perpendicular
to the loop) changes with time.

B(x) = B 0 + bx
+X
v

(a) This problem should be solved with Faradays Law:


E =

d
.
dt

The area of the loop remains constant, but the magnetic


flux will change with time because the loop moves towards
a weaker field.
= A (B0 + bx)
Using the following equation, we can get the time derivative
of the flux at the location x of the loop:
d
d dx
=
= (Ab)(v) = Abv.
dt
dx dt
Now the emf can be calculated using
d
= Abv.
E =
dt
(b) The direction of the current can be determined with
Lenzs Law. The direction of the induced current will be
in a direction to oppose the change in flux. Because the flux
through the loop is decreasing, the induced current will induce a magnetic field inside the loop pointing to the right.
This means, as viewed from the origin (on the left), the current will be going in a clockwise direction (CW).
(c) If the coil is moving from left to right instead, the problem
is similar to part (a), except the velocity is positive, and so
the answer is Abv. Since no sign convention is defined for
the emf enter Abv for Mastering Physics.
(d) Because the coil is moving in the opposite direction compared to parts (a) and (b), the flux through the loop is increasing, and so the induced current will induce a magnetic
field pointing towards from the origin. So, viewed from the
origin (on the left), the current will be in a counter-clockwise
direction (CCW), the opposite direction from part (b).

The angle between the magnetic field B and the vector A


is given by = t. Thus d/dt = , and the flux through
the coil at any given time can be written as,
B = B A = BA cos = BA cos t.
From Faradays law, the induced emf is given by the change
of the flux with respect to time. But here, we must remember
we have a coil with N = 120 turns, thus,
E = N

dB
= N BA sin t
dt

The maximum induced emf occurs when sin t = 1. We can


solve for the angular speed that would create the stated emf:
Emax = N BA
2.60 102 V
rad
Emax
=
= 7.31
=
N BA
120(7.80 102 T)(0.0195 m)2
s

29-17 Consider the system shown below. (a) Using Lenzs


law, determine the direction of the current in resistor ab of
the figure when switch S is opened after having been closed
for several minutes. (b) Using Lenzs law, determine the
direction of the current in resistor ab of the figure when coil
B is brought closer to coil A with the switch closed. (c)
Using Lenzs law, determine the direction of the current in
resistor ab of the figure when the resistance of R is decreased
while the switch remains closed.

29-20 A 1.60 m long metal bar is pulled to the right at a


steady 4.5 m/s perpendicular to a uniform, 0.755 T magnetic
field. The bar rides on parallel metal rails connected through
R = 26.0 , as shown in the figure, so the apparatus makes
a complete circuit. You can ignore the resistance of the bar
and the rails. (a) Calculate the magnitude of the emf induced
in the circuit. (b) Find the direction of the current induced
in the circuit. (c) Calculate the current through the resistor.

(a) The current will flow from the positive terminal of the
battery when S is closed. Thus, the current flowing in coil
A will produce a magnetic field inside coil A pointing to
the right (and also to the right inside coil B). When S is
then opened, the current will stop flowing and the magnetic
field it produces will decrease. Lenzs Law says that the E
induced in the coil B will oppose the change in magnetic flux
by causing current that will create an additional magnetic
field to the right. Thus, by the right hand rule the current
in coil B will flow from a to b.
(b) When S is closed, coil A produces a magnetic field inside
the coils pointing to the right. As coil B is brought closer to
A, the magnetic field inside coil B becomes stronger, so the
flux becomes larger. The E induced in coil B will oppose the
change in magnetic flux by causing current that will create
additional magnetic field to the left. By the right hand rule,
the current in coil B will flow from b to a.
(c) If the resistance in R is decreased, the relation I = V /R
tells us that the current will increase, since the battery voltage V does not change. When the current through coil A
increases, the magnetic field directed to the right will also
increase. The E induced in coil B will oppose the change in
magnetic flux by causing current that will create additional
magnetic field to the left. By the right hand rule, the current
in coil B will flow from b to a. [This argument is exactly the
same as the one used in part (b)].

The flux at any time is the area of the loop times the magnitude of the magnetic field B, and we let L be the length
of the bar. In a time t the area of the circuit will increase
by Lx, where x and t are related to the speed of the
bar by v = x/t.
(a) The emf is given by
E = /t = BLx/t = BLv
= 0.755 T 1.60 m 4.5 m/s = 5.44 V
(b) Pulling the rod increases the flux by adding magnetic
field directed into the page. By Lenz Law, the induced
current must then produce a magnetic field inside the loop
directed out of the page. The current must therefore be
counterclockwise.
(c) By Ohms Law,
E = IR I = 5.44 V/26.0 = 0.21 A

Physics 21
Fall, 2011

Solution to HW-18

30-7 At the instant when the current in an inductor is increasing at a rate of 6.45 102 A/s, the magnitude of the
self-induced emf is 1.65 102 V. (a) What is the inductance of the inductor? (b) If the inductor is a solenoid with
405 turns, what is the average magnetic flux through each
turn when the current is 0.715 A?
(a) In the class lecture we applied Faradays Law to a
B
solenoid with N turns. The self-induced emf is E = N d
dt ,
and we defined the self-inductance L by
E = L

di
dt

Here we arent concerned with the direction of the emf, only


the magnitude. Therefore we can substitute the given values
to obtain
1.65 102 V
E
=
di/dt
6.45 102 A/s
= 0.256 H

L=

(b) For this part we must notice that since the flux through
one loop of a solenoid of length l is given by
B = BA = 0

N
B
N2
A = N 0 A = N
.
l
l
i

Note that the relation above,


L=N

B
,
i

is in fact the definition used in the book for L [Eq. (30.6)].


We can use it to answer this problem by solving for B .
(0.256 H) (0.715 A)
Li
=
N
405
4
= 4.52 10 Wb

B =

30-19 An inductor with an inductance of 2.50 H and a


resistance of 8.00 is connected to the terminals of a battery
with an emf of 6.00 V and negligible internal resistance. Find
(a) the initial rate of increase of current in the circuit; (b) the
rate of increase of current at the instant when the current
is 0.500 A; (c) the current 0.250 s after the circuit is closed;
(d) the final steady-state current.
(a) Applying Kirchhoffs Law, we find the loop equation is
E iR L

E iR
6 V (0.5 A 8 )
di
=
=
= 0.8 A/s.
dt
L
2.5 H
(c) Using the general solution for the loop equation, we have
6V

E
i=
1 e(R/L)t =
1 e(8/2.5 H)0.25 s
R
8
= 0.413 A.
(d) The general solution shows that as t , the exponential term vanishes. Then
i=

di
E iR
di
= 0 =
=
.
dt
dt
L

E
6V
=
= 0.75 A.
R
8

30-29 A 7.60 nF capacitor is charged up to 13.0 V, then


disconnected from the power supply and connected in series
through a coil. The period of oscillation of the circuit is then
measured to be 9.00105 s. (a) Calculate the inductance of
the coil. (b) Calculate the maximum charge on the capacitor.
(c) Calculate the total energy of the circuit. (d) Calculate
the maximum current in the circuit.
(a) The resonant frequency of an LC circuit is related to L,
C, and the period of the oscillations T by

N
iA,
l

we can rewrite the expression for the inductance L given on


the equation sheet in terms of the flux,
L = 0

(b) Using the loop equation again for i = 0.5A,

1
2
=
.
T
LC

(The frequency formula is given on the equation sheet; for


this case R = 0.) Solving for L leads to
L=

(9.00 105 s)2


T2
=
= 2.70 102 H.
4 2 C
4 2 (7.60 109 F)

(b) The capacitor has its maximum charge when it is initially


charged to V = 13 V:
Q = CV = (7.60 109 F)(13 V) = 9.88 108 C.
(c) Energy is conserved, and the total energy of the oscillator
is the energy initally stored in the capacitor:
U = 21 CV 2 = 21 (7.60 109 F)(13 V)2 = 6.42 107 J.
(d) In the LC oscillator, the energy swings back and forth
between the capacitor and the inductor. At one point, the
total energy U of the circuit is all in the inductor. At that
time, the current through the inductor is a maximum, and we
can use the expression for energy in the inductor, U = 21 LI 2 ,
and solve for I:
r
2U
I=
L
s
2(6.42 107 J)
= 6.90 103 A = 6.90 mA.
=
(2.70 102 H)

The initial condition is that i = 0 at t = 0, so at that time


E
6V
di
= =
= 2.4 A/s.
dt
L
2.5 H

October 28, 2011

30-22 In the figure below, switch S1 is closed while switch


S2 is kept open. The inductance is L = 0.115 H, and the
resistance is R = 120 . a) When the current has reached
its final value, the energy stored in the inductor is 0.260 J.
What is the emf E of the battery? b) After the current has
reached its final value, S1 is opened and S2 is closed. How
much time does it take for the energy stored in the inductor
to decrease to half of its original value?

30-33 An LC circuit containing an 86.0 mH inductor and


a 1.25 nF capacitor oscillates with a maximum current of
0.760 A. (a) Calculate the maximum charge on the capacitor.
(b) Calculate the oscillation frequency of the circuit. (c) Assuming the capacitor had its maximum charge at time t = 0,
calculate the energy stored in the inductor after 2.60 ms of
oscillation.
(a) The charge on the capacitor oscillates as a function of
time. Assuming the capacitor starts with the maximum
charge at t = 0 (as stated in part c), then we know what
the charge and current will look like as functions of time:

(a) When the current has reached its final value I0 , the energy stored in the inductor is given by
U0 = 21 LI02 = 12 L

E
R

where we used Ohms Law to express the current in terms


of the emf and resistance. We can rearrange this to solve for
the emf and substitute our known values to find
E=

2 UR
L

1
2

1
2(0.260 J)(120 )2 2
= 255 V.
(0.115 H)

q(t) = Qmax cos(t)


dq
= Qmax sin(t) = Imax sin(t)
i(t) =
dt

where = 1/ LC. The maximum quantities correspond to


the charge and current at times when the cosine and sine
are equal to one, respectively. Since q(t) and i(t) are related by a time derivative, the second equation above gives
a relationship between Qmax and Imax :

Qmax = Imax / = Imax LC


p
= (0.760 A) (0.086 H)(1.25 109 F)
= 7.88 106 C
(b) The oscillation frequency of the circuit is related to
by:
f=

1
p

=
=
2
2 LC
2 (0.086 H)(1.25 109 F)

= 1.54 104 Hz

(b) When the switches are changed, the inductor will release
the energy stored in the magnetic field, and this will lead to
a current that decays exponentially with time constant L/R
(see page 1044 of the text.) The current is given by

(c) The energy stored in the inductor at t = 2.60 ms is given


by

i(t) = I0 exp(Rt/L),

= 21 (0.086 H)(0.0760 A)2

sin2 2(1.54 104 hz)(0.0026 s)

where I0 is the steady state current from part (a). We can


use this result to determine the energy stored in the inductor
as a function of time:
U (t) = 21 Li(t)2 = 12 LI02 exp(2Rt/L) = U0 exp(2Rt/L).
We want to find the time t when the inductor has half of its
initial energy. At that time,
exp(2Rt/L) = 21 .
Solving this equation for t we find
t=

L
0.115 H
ln 2 =
ln 2 = 0.332 ms.
2R
2 (120 )

U (t ) = 21 L[i(t )]2
2
= 12 LImax
sin2 (2f t )

= 7.03 103 J

Physics 21
Fall, 2011

Solution to HW-19

29-34 A dielectric of permittivity 3.3 1011 F/m completely fills the volume between two capacitor plates. For t >
0 the electric flux through the dielectric is (7800 V m/s3 )t3 .
The dielectric is ideal and nonmagnetic; the conduction current in the dielectric is zero. At what time does the displacement current in the dielectric equal 23 A?

29-51 As a new electrical engineer for the local power company, you are assigned the project of designing a generator
of sinusoidal ac voltage with a maximum voltage of 120 V.
Besides plenty of wire, you have two strong magnets that
can produce a constant uniform magnetic field of 1.8 T over
a square area with a length of 10.4 cm on a side when the
magnets are separated by a distance of 12.1 cm. The basic
design should consist of a square coil turning in the uniform
magnetic field. To have an acceptable coil resistance, the
coil can have at most 450 loops.

For a charging capacitor we have the equation for displaceE


ment current iD = 0 d
dt . When a dielectric is between the
capacitor plates, we multiply 0 by the dielectric constant K
to get the permittivity of the material = K0 . Thus we
E
have iD = d
dt . The function for the electric flux can then
be substituted:
3

d
3 3
3 d t
(7800 V m/s )t = (7800 V m/s )
iD =
dt
dt
= (7800 V m/s3 )(3)t2
Solving this for time:
s
iD
t=
(7800 V m/s3 )(3)
s
23 106
= 5.5 s
=
3.3 1011 (7800 V m/s3 )(3)

We can use Faradays law to calculate the induced EMF.


I
Z
d
d
E dl =
B dA = ,
dt
dt
The flux through a square loop perpendicular to the magnetic field with the maximum possible dimensions and N
loops of wire will be B = N BA, where A is the area. If
the loop is rotated within the magnetic field, then the flux
through the loop will change because number of field lines
going through the loop will be changing. Rotating at an
angular frequency of , the flux will change with time:
B (t) = N BA cos(t)
The time derivative of this gives the induced EMF:
E =

dB
= N BA sin(t)
dt

The amplitude is then N BA. Solving for


=

120 V
E
= 13.7 rad/sec
=
N BA
450 1.8 .1042

Mastering physics wants an answer in revolutions per


minute, so the conversion is:
13.7

rad 60 sec 1 rev

= 130.8 rpm
sec 1 min 2 rad

November 4, 2011

31-6 A capacitance C and an inductance L are operated at


the same angular frequency. (a) At what angular frequency
will they have the same reactance? (b) If L = 4.80 mH
and C = 3.70F, what is the numerical value of the angular
frequency in part (a)? (c) What is the reactance of each
element?
(a) Equate the reactances and solve for :
1
= L
C

1 = 2 LC

1
=
.
LC

(b) Substituting numbers,


=p

1
(4.80 103 )(3.70 106 )

31-14 You have a 180 resistor and a 0.430 H inductor.


Suppose you take the resistor and inductor and make a series
circuit with a voltage source that has a voltage amplitude of
28.0 V and an angular frequency of 250 rad/s. (a) What is
the impedance of the circuit? (b) What is the current amplitude? (c) What is the voltage amplitude across the resistor?
(d) What is the voltage amplitudes across the inductor? (e)
What is the phase angle of the source voltage with respect
to the current? (f) Does the source voltage lag or lead the
current? (g) Construct a phasor diagram.
(a) The impedance relates the peak values of the voltage and
current. We can find it by drawing the phasor diagram:
VL = LI

= 7.50 103 rad/s.

V = ZI

(c) Again substituting numbers,


1
= L = 36.0 .
C
31-10 You want the current amplitude through an inductor
with an inductance of 5.00 mH (part of the circuitry for a
radio receiver) to be 2.10 mA when a sinusoidal voltage with
an amplitude of 12.0 V is applied across the inductor. (a)
What frequency is required?
(a) The frequency f is related to angular frequency by the
equation = 2f . The amplitude of the voltage across the
inductor is VL = IL = IL2f . Thus the frequency is
12.0V
VL
=
f=
3
2IL
2(2.10 10 A)(5.00 103 H)
= 182 kHz

VR = RI

The diagram is simpler since there is no capacitor. The


voltage V of the power supply, shown by the dashed line, is
p
p
V = (LI)2 + (RI)2 = (L)2 + R2 I = ZI.
Therefore the impedance Z is given by
p
Z = (180 )2 + (250 rad/s 0.430 H)2 = 210 .

(b) Weve already written the relation between V and I, so


I=

28.0 V
V
=
= 0.134 A = 134 mA.
Z
210

(c) To find the voltage amplitude across the resistor we use


Ohms law and the current from part (b):
VR = IR = (0.134 A)(200 ) = 24.0 V
(d) The voltage amplitude across the inductor is the inductive reactance XL times the current:
VL = LI = (250 rad/s)(0.430 H)(.134 A) = 14.4 V
(e) Using the values of VL and VR just calculated, we can
easily find the phase angle from the diagram above:
= arctan

14.4V
VL
= arctan
= 30.8 .
VR
24.0V

(f) From the phasor diagram above it is clear that the power
supply voltage V leads the current. (Remember the current
is in phase with VR ).
(g) The diagram is shown above.

Physics 21
Fall, 2011

Solution to HW-20

31-21 You have a 207 resistor, a 0.408 H inductor, a


4.99 F capacitor, and a variable-frequency ac source with
an amplitude of 2.93 V. You connect all four elements together to form a series circuit. (a) At what frequency will
the current in the circuit be greatest? (b) What will be
the current amplitude at this frequency? (c) What will be
the current amplitude at an angular frequency of 410 rad/s?
(d) At this frequency, will the source voltage lead or lag the
current?

31-23 In an LRC series circuit, the rms voltage across the


resistor is 30.9 V, across the capacitor is 89.1 V, and across
the inductor is 50.2 V. What is the rms voltage of the source?
Since this is a series circuit (there is only one loop), the
voltage supplied by the source will be equal to the sum of the
instantaneous voltages across all the other circuit elements.
However we cannot just add the rms voltages together since
they are all not in phase with each other. We can use a
phasor diagram to add the voltages together vectorially.

(a) We need to look at the relationship V = IZ. Since V


is fixed, in order to maximize I we need to minimize Z. Z
depends on via XL and XC :
q
2
Z = R2 + (XL XC )
Clearly, Z is minimized when XL XC = 0. Using the
definitions of XL and XC ,
1
= L
C
1
2 =
LC
1
1
= 701 rad/s
=
=p
LC
(0.408 H) (4.99 F)
(b) When XL XC = 0, Z = R, so
I=

2.93 V
V
=
= 14.2 103 A
Z
207

From this we see we can use the Pythagorean theorem to


determine the total voltage across all three circuit elements.
p
V = (VR )2 + (VL VC )2
p
= (30.9 V)2 + (50.2 V 89.1 V)2 = 49.7 V
V is the vector sum of the rms voltages across all the
circuit elements, so the rms source voltage is V0 = 49.7 V.

(c) Now,
XL XC = (410 rad/s) (0.408 H)

1
(410 rad/s) (4.99 F)

= 322
I=

V
=q
Z

2.93 V
2

(207 ) + (322 )

= 7.66 103 A

(d) Since our calculation of XL XC in part (c) came out


negative, the circuit is mostly capacitive. This means that
voltage lags current (or, equivalently, current leads voltage)
at the power supply.

November 4, 2011

31-28 An L-R-C series circuit is connected to a 120 Hz ac


source that has Vrms = 89.0 V. The circuit has a resistance of
74.0 and an impedance at this frequency of 120 . What
average power is delivered to the circuit by the source?
The average power in an ac circuit is given by
Pav = Vrms Irms cos
In an L-R-C circuit, the root-mean-square current is given
by
Vrms
89.0 V
Irms =
=
= 0.742 A
Z
120
and the power factor is given by
cos =

74.0
R
=
= 0.617
Z
120

Thus, we can solve the average power as


Pav = Vrms Irms cos = (89.0 V)(0.742 A)(0.617) = 40.7 W
31-59 In an LRC series circuit the magnitude of the phase
angle is 50.3 with the source voltage lagging the current.
The reactance of the capacitor is 348 , and the resistor
resistance is 182 . The average power delivered by the
source is 135 W. (a) Find the reactance of the inductor. (b)
Find the rms current. (c) Find the rms voltage of the source.

LI

31-37 A transformer connected to a 120 V ac line is to


supply 12.0 Vrms to a portable electronic device. The load
resistance in the secondary is 4.40 . (a) What should the
ratio of primary to secondary turns of the transformer be?
(b) What rms current must the secondary supply? (c) What
average power is delivered to the load? (d) What resistance
connected directly across the source line (which has a voltage
of 120 V) would draw the same power as the transformer?
(a) The ratio of the number of primary to secondary turns
is equal to the ratio of the voltages of the primary to the
secondary part of the transformer. So,
Np
Vp
120 V
=
=
= 10.
Ns
Vs
12 V
(b) We can determine the rms current supplied since we
know the load resistance and the rms voltage supplied by
the transformer.
Irms =

12 V
Vrms
=
= 2.73 A
R
4.4

(c) We calculate the average power delivered to the load by


using:
Pav = Irms Vrms = (2.73 A)(12 V) = 32.7 W
(d) We want the power on both ends of the transformer to
be the same, so we can calculate the resistance that would
draw this power since we know:
P = IV =

RI = 182 I

V2
R

Solving for R and substituting the power found in part (c)


and the source voltage we find:

= -50.3o
V

R=

I
= 348 I
C
(a) From the diagram we see that
tan(50.3 ) =

LI 348I
L 348
=
.
182I
182

We can easily solve for the reactance of the inductor:


L = 129 .
(b) The rms current is related to the average power by
p
p
Irms = Pav /R = 135 W/182 = 0.861 A
(c) Now use
Pav = 135 W = Irms Vrms cos .
We know everything in this equation except Vrms , so we solve
to get
135 W
= 245 V
Vrms =
(0.861 A) cos(50.3 )

(120 V)2
V2
=
= 440
P
32.7 W

Physics 21
Fall, 2011

Solution to HW-22

K20-7 The wave speed on a string under tension is


170 m/s. What is the speed if the tension is doubled?
From Eq. (3) in the waves handout,

v = T /,
so if the tension T is doubled, v increases by

2.

K20-11 A wave travels with speed 180 m/s. Its wave number is 1.40 rad/m. What is its wavelength? What is its
frequency?

k = 2/ = 2/k = 2/1.40 = 4.49 m


v = f f = v/ = 180/4.49 = 40.1 Hz
K20-42 This is a snapshot graph at t = 0 of a 5.0 Hz wave
traveling to the left. What are the speed and phase constant
of the wave?

K20-52 Earthquakes are essentially sound waves traveling through the earth. They are called seismic waves.
Because the earth is solid, it can support both longitudinal and transverse seismic waves. These travel at dierent
speeds. The speed of longitudinal waves, called P waves,
is 8000 m/s. Transverse waves, called S waves, travel at a
slower 4500 m/s. A seismograph records the two waves from
a distant earthquake. If the S wave arrives 2.0 min after
the P wave, how far away was the earthquake? You can assume that the waves travel in straight lines, although actual
seismic waves follow more complex routes.
Let d be the distance from the epicenter of the earthquake
to the detector. Then the travel time for the P wave is
tP = d/vP and the travel time for the S wave is tS = d/vS .
Then tS tP = t is the delay time for the S wave, and
d
d

= t
vS
vP


1
1
d

= t
vS
vP


vP v S
= t
d
vS vP
vS v P
t
d=
vP v S
Substituting numbers (2.0 min = 120 s) gives
d=

4500 8000
120 = 1230 km.
8000 4500

15-20 A piano wire with mass 2.90 g and length 75.0 cm


is stretched with a tension of 30.0 N. A wave with frequency 110 Hz and amplitude 1.50 mm travels along the
wire. (a) Calculate the average power carried by the wave.
(b) What happens to the average power if the wave amplitude is halved?

D(x, t) = A sin(kx + t + ).
From the graph, = 2.0 m, and f = 5.0 Hz is given. Hence
v = f = 10 m/s.
We already know the graph is at t = 0. At x = 0, the
amplitude is abut 0.5. Since the maximum amplitude on the
graph is 1, we have

0.5 = 1.0 sin(k0 + 0 + ) = sin = 30 .

(a) The average power carried by the wave is given by the


formula P  = 12 A2 2 v, where is the mass per unit length,
and A is the waves amplitude.
 We need to calculate the
velocity of the wave with v = T /, where T is the tension
(the equation sheet uses for the mass per unit length):


30.0
v = T / =
= 88.1 m/s.
(2.9 103 ) /0.75
Using = m/L we can calculate the average power
P  =

1 .0029
(.0015)2 (2 110)2 (88.1) = 0.183 W.
2 .75

(b) If the wave amplitude is halved, then the average power


 2
will change by a factor of 12 = 14 because power depends
on the square of the amplitude.
0.183

1
= .0458 W = 4.58 102 W.
4

November 11, 2011

15-24 A fellow student of mathematical bent tells you that


the wave function of a traveling wave on a thin rope is
D(x, t) = (2.33 mm) cos[(6.98 rad/m)x + (742 rad/s)t]. Being more practical-minded, you measure the rope to have a
length of 1.35 m and a mass of 3.38 g. Determine (a) the amplitude, (b) the frequency, (c) the wavelength, (d) the wave
speed, (e) the direction the wave is traveling, (f) the tension
in the rope, and (g) the average power transmitted by the
wave.
(a) Comparing the above equation with the general form
given in class,
D(x, t) = A sin(kx t + ),
one sees that the amplitude is A = 2.33 mm.
(b) the frequency is given by
f=

742 rad/s

=
= 118 Hz
2
2

(c) The wavelength is given by


=

2
2
=
= 0.90 m.
k
6.98 rad/m

(d) The wave speed is


v=

742 rad/s
=
= 106 m/s.
k
6.98 rad/m

(e) The wave travels in the x direction since the argument


has the form of kx + t.
(f) We get the tension T from

v = T / T = v 2
The linear mass density is
=

3.38 103 kg
mass
=
= 2.504 103 kg/m.
length
1.35 m

so
T = v 2 = (2.504 103 kg/m)(106.3 m/s)2 = 28.3 N
(g) The average power transmitted by the wave is given by:

1 2 2
1
1
A v = A2 2 T / =
T A2 2
2
2
2
1
=
(2.50 103 )(28.3) (2.33 103 )2 (742)2
2
= 0.39 W

P  =

Physics 21
Fall, 2011

Solution to HW-23

K 20-53 One way to monitor global warming is to measure


the average temperature of the ocean. Researchers are doing
this by measuring the time it takes sound pulses to travel underwater over large distances. At a depth of 1000 m, where
ocean temperatures hold steady near 4 C, the average sound
speed is 1480 m/s. Its known from laboratory measurements
that the sound speed increases 4.0 m/s for every 1.0 C increase in temperature. In one experiment, where sounds
generated near California are detected in the South Pacic,
the sound waves travel 7700 km. If the smallest time change
that can be reliably detected is 1.0 s, what is the smallest
change in average temperature that can be measured?
Let D be the distance the waves travel and v be their velocity. Then their travel time is t = D/v.
D
dt
D
= 2 t 2 v,
dv
v
v
where we dropped the minus sign. We can relate the changes
in velocity and temperature:
v
4.0 m/s,
T

so v (4.0 m/s)T.

Substituting the expression for v into the expression above


for t, we obtain
t

YF 16-75 The sound source for a ships sonar system operates at a frequency f = 23.0 kHz. The speed of sound in
water (assumed to be uniform at 20 C) is 1482 m/s. What
is the wavelength of the waves emitted by the source?
We know the velocity of a wave is given by:
v = f
We know the frequency of the speed of sound as it propagates
through water so we can determine the wavelength emitted.
=

1482 ms
v
=
= 0.0644 m
f
23000 Hz

16-38 Two guitarists attempt to play the same note of


wavelength 6.50 cm at the same time, but one of the instruments is slightly out of tune and plays a note of wavelength
6.51 cm instead. What is the frequency of the beat these
musicians hear when they play together?
Beats are heard when two tones with slightly dierent frequencies fa and fb are sounded together. fbeat = fa fb .
We also need to know that the speed of a wave is equal to
the product of wavelength and frequency, v = f . Thus,




1
1
1
1

fbeat = v

= 344 m/s
a
b
6.50 cm 6.51 cm
= 8.0Hz

D
v2
t.
(4.0 m/s)T. T
2
v
D(4.0 m/s)

Substituting t = 1 s, v = 1480 m/s, and D = 7700 km gives


T 7.11 102 C.
K 21-40 A violinist places her nger so that the vibrating
section of a 1.10 g/m string has a length of 30.0 cm, then
she draws her bow across it. A listener nearby hears a note
with a wavelength of 60.0 cm. Take the speed of sound in
air to be 343 m/s. What is the tension in the string?
The key point is that the sound in air will have the same
frequency as the vibrating violin string. Since in general
f = v, we have
f=

343 m/s
vair
= 572 Hz
=
air
0.6 m

Substituting numbers,
T = (1.10 103 kg/m)(2 0.3 m 572 Hz)2 = 130 N.

November 18, 2011

YF 16-67 A platinum wire (density = 21.4 g/cm ) has


diameter d = 215 m and length L = 0.500 m. One end
of the wire is attached to the ceiling, while a mass m =
440 g is attached to the other end so that the wire hangs
vertically under tension. If a vibrating tuning fork of just
the right frequency is held next to the wire, the wire begins
to vibrate as well. What tuning-fork frequencies will cause
this to happen? You may assume that the bottom end of the
wire (to which the mass is attached) is essentially stationary,
and that the tension in the wire is essentially constant along
its length.

K 20-50 One cue your hearing system uses to localize a


sound (i.e., to tell where a sound is coming from) is the
slight dierence in the arrival times of the sound at your ears.
Your ears are spaced approximately 20 cm apart. Consider a
sound source 5.0 m from the center of your head along a line
45 to your right. What is the dierence in arrival times?
Give your answer in microseconds.

(a) The problem here is to nd the vibrational frequencies of


a stretched wire held xed at both ends. The wire is under
tension T = mg because of the gravitational force acting
on the weight. Both ends of the wire are xed: one end is
attached to the ceiling, and a heavy weight is at the other
end. The fundamental mode will have a wavelength
= 2L
and a frequency f that satises f = v, where v = T /,
and is the mass per unit length of the wire. We nd by
dividing the total mass of the wire by its length:
=

volume density
(d2 /4)L
d2
mass
=
=
=
.
L
L
L
4

Putting all these pieces together, we have





T /
4mg/(d2 )
v
1
mg
f= =
=
=

2L
2L
Ld

(0.440 kg)(9.80 m/s2 )
1
=
(0.5 m)(0.000215 m)
21400 kg/m3
= 74.5 Hz.
Any positive integer multiple of this frequency is a vibrational frequency of the wire.

To convert to s, multiply the number of seconds by 106 .


The result is 412 s.

Physics 21
Fall, 2011

Solution to HW-24

16-33 Two loudspeakers, A and B, are driven by the same


amplier and emit sinusoidal waves in phase. Speaker B is
2.00 m to the right of speaker A. Consider point Q along
the extension of the line connecting the speakers, 1.00 m to
the right of speaker B. Both speakers emit sound waves that
travel directly from the speaker to point Q. (a) What is the
lowest frequency for which constructive interference occurs
at point Q? (b) What is the lowest frequency for which
destructive interference occurs at point Q?

16-70 Two identical loudspeakers driven by the same amplier are located at points A and B, 2.00 m apart. The
frequency is 784 Hz and the speed of sound in air is 344
m/s. A small microphone is moved out from point B along
a line perpendicular to the line connecting A and B (line BC
in the gure). At what distances from B will there be (a) destructive interference and (b) constructive interference? (c)
If the frequency is made low enough, there will be no positions along line BC at which destructive interference occurs.
How low must the frequency be for this to be the case?
A
LA
2.0 m

Speaker A is 3.0 m from Q, and speaker B is 1.0 m from


Q, so the path dierence is 2.0 m. (a) The speakers are in
phase, so for constructive interference, the path dierence
must be an integral number of wavelengths (n). We choose
n = 1 to get the lowest frequency, so = 2.0 m. Hence
f = v f =

343 m/s
v
=
= 172 Hz

2.0 m

(b) For destructive interference the path dierence must be


0.5 wavelengths. [(n+ 12 ) gives destructive interference, and
lowest frequency is from n = 0.] Hence take = 4.00 m and
f=

v
343 m/s
=
= 86 Hz

4.0 m

LB = x

(a) There will be destructive interference when the dierence


of the path lengths from each speaker tothe microphone
is

a half integer wavelength, or LA LB = n + 12 , where n
is an integer. To calculate the wavelength , use
=

344 m/s
v
=
= 0.439 m.
f
784 s1

The condition for destructive interference is



LA LB = x2 + 2.02 x = (n + 12 ).
Now solve the equation above for x:

x2 + 2.02 = x + (n + 12 )

x2 + 4 = x2 + (2n + 1)x + (n + 12 )2 2

x=

4 (n + 12 )2 2
2(n + 12 )

Substituting n=0, 1, 2, 3, 4, we get x0 =0.026 m, x1 =0.53 m,


x2 =1.27 m, x3 =2.71 m, and x4 =9.0 m.
(b) For constructive interference, LA LB must be an integer
number of wavelengths, n. A similar analysis then gives
4 n2 2
.
2n
Evaluating x for several n gives x0 =0.26 m, x1 =0.86 m,
x2 =1.8 m, and x3 =4.3 m.
(c) As the frequency decreases, the wavelength increases, and
the mimimum value LA LB = 12 needed for destructive
interference
increases. By calculating several values of LA

LB + x2 + 2.02 x, you will nd that the maximum value


of LA LB is 2.0 m at x = 0, and that LA LB gets smaller
and smaller as x increases. For any frequency such that 12 is
greater than 2.0 m, there will be no positions where LA LB
is 2.0 m. Hence at the cuto frequency fmin , 12 = 2 m or
= 4 m. We solve for this frequency using
x=

fmin =

344 m/s
v
=
= 86 Hz.

4m

November 18, 2011

32-11 Radio station WCCO in Minneapolis broadcasts at


a frequency of 830 kHz. At a point some distance from the
transmitter, the magnetic eld amplitude of the electromagnetic wave from WCCO is 4.92 1011 T. (a) Find the
wavelength. (b) Find the wave number. (c) Calculate the
angular frequency. (d) Calculate the electric eld amplitude.
(a) For any wave, v = f , and for a radio wave v = c. Hence
=

3.00 108 m/s


c
=
= 361 m.
f
8.30 105 Hz

(b) Since we now have the wavelength, , we can use


k=

2
= 1.74 102 m1 .

(c) We were given the frequency of the wave, f , so to nd


the angular frequency, , we use
= 2f = 5.22 106 rad/s.
(d) For an electromagnetic wave, the amplitude of the electric eld is c times the amplitude of the magnetic eld:
E0 = cB0 = (3.00 108 m/s)(4.92 1011 T)
= 1.48 102 V/m.
32-16 Consider each of the electric- and magnetic-eld orientations. (a) What is the direction of propagation of the
wave if E = E i, B = B j. (b) What is the direction of
propagation of the wave if E = E j, B = B i. (c) What

is the direction of propagation of the wave if E = E k,


B = B i. (d) What is the direction of propagation of the
In each case, express the diwave if E = E i, B = B k.
rection of the propagation vector as a unit vector. Its three
components should be entered as x, y, z, separated by commas. For example, if the wave propagates only in the x
direction, enter 1, 0, 0.
The direction of propagation is the direction of E B. By
remembering cyclic order, its easy to evaluate the product
j k
= i, and k
i = j.
of any two unit vectors: i j = k,

(a) The direction is i (j) = (i j) = k. Enter 0, 0, 1.


Enter 0, 0, 1.
(b) The direction is j i = k.
(i) = k
i = j. Enter 0, 1, 0.
(c) The direction is (k)

= j. Enter 0, 1, 0.
(d) The direction is i (k) = (i k)

Physics 21
Fall, 2011

Solution to HW-25

32-15 We can reasonably model a 60 W incandescent lightbulb as a sphere 5.2 cm in diameter. Typically, only about
5% of the energy goes to visible light; the rest goes largely
to nonvisible infrared radiation. a) What is the visible light
intensity at the surface of the bulb? b) What is the amplitude of the electric field at this surface, for a sinusoidal
wave with this intensity? c) What is the amplitude of the
magnetic field at this surface, for a sinusoidal wave with this
intensity?
(a) The intensity of light is given by the power per unit area,
so the visible light intensity Ivis at the surface of the light
bulb is the power due to visible light divided by the surface
area of the light bulb:
Ivis =

Pvis
0.05Ptot
(0.05)(60.0)
=
=
= 353 W/m2 .
A
4(d/2)2
4(0.026)2

(b) The intensity at the surface of the bulb calculated above


is just the magnitude S of the time averaged Poynting vector,
which is related to the electric field amplitude E0 by
S = 12 0 cE02 .
Using this expression we can solve for the amplitude of the
electric field:

2S
2Ivis
E0 =
=
= 516 V/m.
0 c
0 c
(c) The amplitudes of the electric and magnetic fields are
related by
B0 =

E0
= 1.7 T.
c

We could have found the magnetic field amplitude directly

from S using an alternate expression for S:

c
20 S
S = 12 B02 = B0 =
.
0
c

32-41 A small helium-neon laser emits red visible light


with a power of 3.40 mW in a beam that has a diameter
of 2.00 mm. (a) What is the amplitude of the electric field
of the light? (b) What is the amplitude of the magnetic
field of the light? (c) What is the average energy density
associated with the electric field? (d) What is the average
energy density associated with the magnetic field? (e) What
is the total energy contained in a 1.00 m length of the beam?
(a) The amplitude of the electric field of the light is related
to the time average of the Poynting vector S by
1
S = 0 cE02 .
2
The time average of the Poynting vector is in units of power
per unit area; it can be calculated by dividing the power
given by the area of the beam:
.0034
S =
W/m2 = 1082 W/m2 .
0.0012
Solving the above equation for electric field yields

2S
2 1082.
=
= 903 V/m
E0 =
0 c
0 c
(b) One can find the amplitude of the magnetic field in a
similar manner, using another relation from the equation
sheet:

0
1
2S
c
2
S =
B0 = B0 =
= 3.01 106 T.
2 0
c
One could also use B = E/c.
(c) The energy density associated with the electric field is
uelec = 12 0 E 2 .
However, this expression gives the instantaneous energy density at the time when the electric field magnitude is E. To
find the average energy density associated with the electric field, you must use the time-averaged value of E 2 . Because the electric field is sinusoidal, E(t) = E0 cos(t) and
E(t)2 = E02 cos2 (t). The time average of cos2 t = 21 , so
huelec i = 12 0 hE 2 i = 41 0 E02 = 1.80 106 J/m3
(d) Similarily, the average density associated with the magnetic field is
1 2
1
hB 2 i =
B = 1.80 106 J/m3 .
humag i =
20
40 0
Note that the average energy in the electric field is the same
as the average energy in the magnetic field.
(e) To find the total energy contained in a 1.00 m length of
the beam, use the total (average) energy density multiplied
by the volume of that length. The total energy density is
hutot i = huelec i + humag i = 3.60 106 J/m3
The volume of a 1.00 m length of beam is
V = r2 L = .0012 1.0 = 3.14 106 m3 ,
and the total energy is
hutot iV = 1.13 1011 J.

December 2, 2011

32-18 A sinusoidal electromagnetic wave from a radio station passes perpendicularly through an open window that
has area of 0.500 m2 . At the window, the electric field of the
wave has an rms value 2.60 102 V/m. How much energy
does this wave carry through the window during a 30.0 s
commercial?
The Poynting vector gives the energy per unit area per unit
time carried by the electromagnetic wave. To get the energy
carried through the window in 30 s, we just multiply the
magnitude S of the time-averaged Poynting vector by the
area of the window and then by the length of the time interval. (Since the wave propagates perpendicular
to the win
dow, the surface integral over the window is S dA = SA.)

S is given on the equation sheet in terms of the electric field


amplitude by

The fact that we get a negative result means that the image
is inverted. Mastering Physics only asks for the size of the
image in this part, so enter the absolute value of the result.
(c) The image is inverted.
(d) The fact that we got a positive result in part (a) means
that the image is in front of the mirror and it is a real image.
(This image could be focussed on a screen.)
(e) Here is the ray diagram for this situation:

S = 12 0 cE02 .
Since we are given Erms , it is convenient to
replace the peak
amplitude E0 by Erms , using Erms = E0 / 2. The result is
2
.
S = 0 cErms

The energy is then


2

E = SAt
= 0 cErms
At
12
= (8.85 10 )(3.00 108 )(0.026)2 (0.500)(30.0)
= 2.69 105 J.

34-5 An object 0.550 cm tall is placed 17.0 cm to the left


of the vertex of a concave spherical mirror having a radius
of curvature of 22.0 cm. (a) Determine the position of the
image. (b) Determine the size of the image. (c) Determine
the orientation of the image. (d) Determine the nature (real
or virtual) of the image. (e) Make a ray diagram and bring
it to recitation.
Since we are dealing with mirrors, the equations we will use
are
1
1
1
+
=
do
di
f

and

m=

hi
di
=
ho
do

The focal length f is positive since the mirror is convex, and


f=

R
22.0 cm
=
= 11.0 cm
2
2

We take do = 17.0 cm to be positive since it is in front of


the mirror. We take ho = 0.550 cm to be positive since the
object is always assumed to be upright.
(a)
1
1
1
=
=
di
f
do
f do
11.0 cm 17.0 cm
di =
=
= 31.2 cm
do f
17.0 cm 11.0 cm
(b)
hi = ho

di
31.2 cm
= 0.550 cm
= 1.01 cm
do
17.0 cm

YF 34-8 An object is a distance of 25.0 cm from the CENTER of a silvered spherical glass Christmas tree ornament
which has a diameter of 5.70 cm. (a) What is the position of
its image? Use the mirror equation to answer this question,
but draw a ray diagram and bring it with you to recitation.
Hint: be careful to determine do (the distance to the SURFACE of the mirror) correctly. (b) What is the magnification
of its image?

YF 34-14 A spherical, concave, shaving mirror has a radius of curvature of 32.5 cm. (a) What is the magnification
of a persons face when it is a distance do = 11.6 cm from
the center of the mirror? (b) What is the distance di to
the image? (c) Is the image real or virtual? Use the mirror
equation to answer this question, but draw a ray diagram
and bring it with you to recitation.

Work part (b) first:


(b) The focal length is half the radius of curvature, or
16.25 cm. (distances in cm.) The sign is positive for a
concave mirror.
1
1
1
1
1
1
=
=

=
di
f
do
16.25 11.6
40.5
The image is 40.5 cm behind the mirror.
(a) The magnification is
m=

di
40.5
=
= 3.49.
do
11.6

(c) The image is virtual.

Physics 21
Fall, 2011

Solution to HW-26

34-23 An insect 3.00 mm tall is placed 25.0 cm to the left


of a thin planoconvex lens. The left surface of this lens is
flat, the right surface has a radius of curvature of magnitude
13.0 cm, and the index of refraction of the lens material is
1.70. (a) Calculate the location of the image this lens forms
of the insect. (b) Calculate the size of the image. (c) Is the
image real or virtual? Erect or inverted?

34-28 A photographic slide is to the left of a lens. The lens


projects an image of the slide onto a wall 6.00 m to the right
of the slide. The image is 80.0 times the size of the slide. (a)
How far is the slide from the lens? (b) Is the image erect or
inverted? (c) What is the focal length of the lens? (d) Is the
lens converging or diverging?

lens
do

di
do + di = L

(a) The image is on the opposite side of the lens, therefore, di


is positive. Since we know do is also positive, we can conclude
that the magnification m = di /do will be negative. We can
write two equations for do and di from the values of L and
|m| given;
do + di = L
di = |m|do
(a) Using the formula for the focal length of a lens with two
curved surfaces (the Lensmakers Equation), with n = 1.7,
R1 = , and R2 = 13 cm, we have
(
)
(
)
1
1
1
1
1
1
= (n 1)

= 0.7

=
f
R1 R2
13
18.57 cm
The value of f determined can be substituted into the lens
equation:
1
1
1
=
+
f
do
di

600 cm
L
=
= 7.41 cm.
|m| + 1
80 + 1

(b) In the last part we found that m was negative, which


implies an inverted image.
(c) We get the focal length of the lens from the lens equation:
(
)
1
1
1
1
1
1
1
=
+
=
+
=
1+
.
f
do
di
do
|m|do
do
|m|

f=
1

1
18.57

1
25

= 72.2 cm

b) To calculate the size of the image, use the magnification


m=

do =

Inverting both sides of the equation, we find

The result is:


1
1
1
=
+
= di =
18.57
25 di

We solve these two equations by substituting the second into


the first, which leads to

72.2
di
=
= 2.89
do
25.0

The image height hi is


hi = mho = 2.89(3.00 mm) = 8.67mm
(c) The image is real because the image distance is positive.
The image is inverted because the magnification is negative.
(d) If the lens is reversed, then R1 = 13 cm and R2 = .
The focal length is therefore:
(
)
1
1
1
1
= (0.7)

=
f
13
18.57 cm
The value of 1/f is exactly the same as we had before. All
the results are the same.

|m|
80
do =
7.41 cm = 7.32 cm
|m| + 1
80 + 1

(d) The focal length f is positive, so the lens is converging.


33-9 Light traveling in air is incident on the surface of a
block of plastic at an angle of 61.3 to the normal and is
bent so that it makes a 47.8 angle with the normal in the
plastic. Find the speed of light in the plastic.
The speed of light in the material is given by the equation
v = c/n. Snells Law states
nair sin(air ) = nplas sin(plas ).
The index of refraction of air is 1.00. Thus the index of
refraction of the plastic is
nplas = sin(air )/ sin(plas ).
Using this in the equation for the speed of the wave we obtain
vplas = c/nplas = c

sin(plas )
sin(47.8 )
= (3.00 108 )
sin(air )
sin(61.3 )

= 2.53 108 m/s.

December 2, 2011

YF 34-33 A diverging lens with a focal length of 47.0 cm


forms a virtual image 8.00 mm tall, 16.5 cm to the right of the
lens. (a) Determine the position of the object. (b) Determine
the size of the object. (c) Is the image erect or inverted? (d)
Are the object and image on the same side or opposite sides
of the lens? Use the lens equation to answer this question,
then draw a ray diagram and bring it to recitation.

ray 2
do

di

ray 3

(a) Since the image is virtual, we can infer that it is on the


same side of the lens as the object. Therefore, its distance di
will be negative. We can calculate the object distance from
the Lens Equation,
1
1
1
1
1
1
=
=

=
do
f
di
47 16.5
25.4 cm
The ray diagram shows the relative positions of the image
and object. (We drew the image and object on the left, since
that is the way weve usually done it in clsss.)
(b) We determine the size of the object from the magnification m = di /do :
hi = mho = ho =

m=

di
8.69
=
= 0.0223
do
390

Then the height of the image is


hi = mho = 0.0223 175 = 3.90 cm = 39.0 mm
(The height is negative because the image is inverted.) Comparing the height of the image to the size of the film, we see
that the image will not fit upright on the film. Of course, you
could tilt the camera and fit the 39
mm image along the films
diagonal, the length of which is 242 + 362 = 43.3 mm.

ray 1

(b) Obtain the image size from the magnification:

hi
8.00 mm
=
= 12.3 mm
m
(16.5/25.4)

(c) From the ray diagram and since m > 0, the image is
upright.
(d) The image and object are on the same side of the lens.

34-32 A converging lens with a focal length of 12.0 cm


forms a virtual image 8.00 mm tall, 17.0 cm to the right of the
lens. (a) Determine the position of the object. (b) Determine
the size of the object. (c) Is the image erect or inverted? (d)
Are the object and image on the same side or opposite sides
of the lens?
(a) Since the image is a virtual image, the sign of di is negative. The position of the object can be found by using the
thin lens equation:
1
1
1
1
1
=
=

= do = 7.03 cm
do
f
di
12 cm 17 cm
(b) The size of the object is related to the size of the image
by the magnification, hi = mho , where m is given by
m=

di
17 cm
0.8 cm
=
= 2.42 ho =
= 0.331 cm.
do
7.03 cm
m

(c) The sign of ho is positive; therefore it is erect. This result


can also be determined by a ray diagram.
(d) The object and image are on the same side of the lens.
In a lens system a virtual image is produced on the same
side as the object that creates the image. The situation is
illustrated by the ray diagram shown below.

34-36 When a camera is focused, the lens is moved away


from or toward the film. If you take a picture of your friend,
who is standing 3.90 m from the lens, using a camera with a
lens with an 85-mm focal length, (a) how far from the film
is the lens? (b) Will the whole image of your friend, who is
175 cm tall, fit on film that is 24 36 mm?

lens
ho
hi
do

di

(a) Use the Lens Equation with do = 390 cm and f = 8.5 cm:
1
1
1
1
1
1
1
1
1
+
=

=
=

=
do
di
f
di
f
do
8.5 390
8.69 cm
The distance di from the film to the lens is 8.69 cm.

The rays are numbered 1, 2, and 3 to correspond to those


described on the optics handout. Notice that on the left
of the lens the rays diverge. This situation occurs when
the object is closer than the focal point. In order to find
the image the rays must be extended back behind the lens
(shown by dotted lines) in order to create a virtual image.

Physics 21
Fall, 2011

Solution to HW-27

33-42 A light ray in air strikes the right-angle prism shown


in the gure (6 B = 28 ). This ray consists of two dierent
wavelengths. When it emerges at face AB, it has been split
into two dierent rays that diverge from each other by 8.50 .
(a) Find the index of refraction of the prism for each of the
two wavelengths.

inner radius, R = 5.02 mm, what is the index of refraction


of the glass? (Hint: Light from the spot on the front surface
is scattered in all directions by the emulsion. Some of it is
then totally reected at the back surface of the plate and
returns to the front surface.)
B R/2
t

R/2

c c
A

The gure above shows the rays that produce the the inner
radius of the halo. A ray leaves from the point source at O,
is totally internally reected (at the critical angle c ) at A,
and then reaches the inner radius of the halo at point B. We
can relate c to the index of refraction n of the glass using
nair
1
= .
n
n
Using the geometry shown in the picture we see that we can
determine c using
( )
R
R/2
1
= c = tan
tan c =
t
2t
sin c =

(a) In order to use Snells Law to nd the index of refraction,


it is necessary to determine the angle each of these light rays
make with a line normal to the surface of the prism. This
means working out a little geometry.
The angle between the incident ray and the normal to the
hypotenuse of the prism is
= 90 28 = 62
Since the 12 angle is given, we can nd the angle between
the upper refracted ray and the normal:

Using R and t given we nd that c = 35 . The index of


refraction of the glass will be
n=

1
= 1.75.
sin c

One can derive a simple expression for n by direct evaluation


of sin c using the sides of the right triangle ABC:

1
R/2
= n = 1 + (2t/R)2 .
= sin c =
n
(R/2)2 + t2

1 = 90 28 + 12 = 74 .
The angle between the normal and the other refracted ray is
2 = 1 + 8.5 = 82.5
Now we can write Snells Law for these situations as
nred sin 62 = nair sin 74

and nblue sin 62 = nair sin 82.5 ,

where nred (nblue ) is the index for the red (blue) light. We
arent told the exact colors, but we expect the light that is
bent less (more) will be more to the red (blue) end of the
spectrum. Now, letting nair = 1 be the index in air, we solve
for each index:

YF 34-57 A telescope is constructed from two lenses with


focal lengths of 90.0 cm and 20.0 cm, and the 90.0 cm lens is
used as the objective. Both the object being viewed and the
nal image are at innity. (a) Find the angular magnication for the telescope. (b) Find the ABSOLUTE VALUE of
the height of the image formed by the objective of a building 60.0 m tall, 3.00 km away. (c) What is the ABSOLUTE
VALUE of the angular size of the nal image as viewed by
an eye very close to the eyepiece? Give your answer in RADIANS.

sin(74 )
nair sin(1 )
=
= 1.09

sin(62 )
sin(62 )
nair sin(2 )
sin(82.5 )
=
=
= 1.12

sin(62 )
sin(62 )

nred =
nblue

33-45 Old photographic plates were made of glass with a


light-sensitive emulsion on the front surface. This emulsion
was somewhat transparent. When a bright point source is
focused on the front of the plate, the developed photograph
will show a halo around the image of the spot. If the glass
plate has a thickness, t = 3.60 mm, and the halos have an

December 8, 2011

34-89 Two thin lenses with focal lengths of magnitude 15.0


cm, the rst diverging and the second converging, are placed
12.00 cm apart. An object 5.00 mm tall is placed 5.00 cm
to the left of the rst (diverging) lens. (a) Where is the
image formed by the rst lens located? (b) How far from
the object is the nal image formed? (c) Is the nal image
real or virtual? (d) What is the height of the nal image?
(e) Is the nal image erect or inverted?

34-91 An object to the left of a lens is imaged by the lens


on a screen 30.0 cm to the right of the lens. When the lens
is moved 4.00 cm to the right, the screen must be moved
4.00 cm to the left to refocus the image. Determine the
focal length of the lens.
For the rst set up, the image distance is di = 30.0 cm and
let the object distance be the variable do .

(a) In a two lens system, the image from the rst lens, serves
as the object of the second lens. The location of the image
formed by the rst lens can be found using do = 5 cm, f =
15 cm, and solving the lens equation for di .
1
1
1
=
+
f
do
di
Then di = 3.75. That means it the image is be 3.75 cm
to the left of the lens, on the same side as object. It is an
upright, virtual image. We know that it is upright because
the magnication is a positive number. We know that it
is virtual because di is a negative number. A ray diagram
conrms this.

(b) To nd the distance from the nal image to the object,


we need to determine the location of the nal image. The
image from the rst lens becomes the object for the second
lens. We need to determine do : because the image from the
rst lens is 3.75 cm to the left of the rst lens, that means it
is 12 cm + 3.75 cm = 15.75 cm from the second lens. Using
do = 15.75 cm and f = +15 cm, solve for di = 315 cm.
This means that the nal image is 315 cm to the right of the
second lens. So the distance between the object and nal
image is 5 cm + 12 cm + 315 cm = 332 cm.

Thus, the equation for the focal length is


1
1
1
1
1
=
.
+
=
+
f
do
di
do
30.0 cm

In the second set up the object distance is increased by


4.00 cm, thus d0o = do + 4.00 cm. The image distance is
decreased by 8.00 cm (4.00 cm from the lens moving to the
right and another 4.00 cm for the screen moving to the left).
Therefore the new image distance is d0i = di 8.00 cm =
30.0 cm 8.00 cm = 22.0 cm.

These values can be used in another equation for the focal


length
1
1
1
1
1
= 0 + 0 =
+
.
f
do
di
do + 4.00 cm 22.0 cm

(c) The nal image is real, because we get a positive image


distance (di ) for the second lens, and we can tell from the
ray diagram.
(d) We can nd the height of the nal image by multiplying
the magnication of each lens.
][
]
[
315
3.75

5.00 mm = 75 mm
hi = m1 m2 ho =
5
15.75
Mastering Physics wants height (absolute value of hi ) in
units of cm, so enter 7.5.
(e) The nal image is inverted, because the total magnication is negative, and the ray diagram conrms this.

(1)

(2)

We can set the expressions for 1/f given by Eqs. (1) and (2)
equal to each other and use the quadratic equation to solve
for do :
1
1
1
1
+
=
+
do
30.0 cm
do + 4.00 cm 22.0 cm
0 = d2o + 4do 330
do = 16.3 cm.
The quadratic equation for do has two roots, but do must
be positive because the object is on the side of the lens from
which the light comes. Therefore we used the plus sign in
the quadratic formula to ensure do > 0. Now do and the
original di can be used to solve for the focal length f :
1
1
1
1
1
=
+
=
+
f = 10.6 cm.
f
do
di
16.3 cm 30.0 cm

35-8 Youngs experiment is performed with light from excited helium atoms ( = 502 nm). Fringes are measured
carefully on a screen 1.20 m away from the double slit, and
the center of the twentieth fringe (not counting the central
bright fringe) is found to be 10.6 mm from the center of the
central bright fringe. What is the separation of the two slits?

35-12 Coherent light with wavelength 400 nm passes


through two very narrow slits that are separated by d =
0.200 mm and the interference pattern is observed on a
screen L = 4.00 m from the slits. (a) What is the width
of the central interference maximum? (b) What is the width
of the rst-order bright fringe?
(a) The position x = x0 of the rst dark spot is given in
terms of the angle it makes with with central bright spot (at
x = 0) by
d sin d

x0
L
= (m + 12 ) = x0 =
,
L
2d

where we invoked small-angle approximations and set m = 0


since we are interested in the central peak. We have
x0 =

(4.00 m)(400 109 m)


= 0.004 m = 4 mm
2(2 104 m)

The width of the central interference maximum will be double the distance from the bright central spot to the rst dark
spot, which is 2x0 = 8.0 mm.
(b) The width of the rst order bright fringe will be the
distance between the rst and second dark spots. We already
know the position of the rst dark spot. All we must do now
is calculate the position of the next dark spot (at x1 , for
m = 1) and nd the dierence (x1 x0 ):
d sin d

3L
x1
= (1 + 12 ) = x1 =
,
L
2d

Clearly x1 = 3x0 = 12 mm, so the width of the rst order


bright fringe is
x1 x0 = 12 mm 4 mm = 8 mm.

Physics 21
Fall, 2011

Solution to HW-28

33-31 Unpolarized light of intensity 26.0 W/cm2 is incident on two polarizing lters. The axis of the rst lter is at
an angle of 24.8 counterclockwise from the vertical (viewed
in the direction the light is traveling) and the axis of the
second lter is at 65.0 counterclockwise from the vertical.
What is the intensity of the light after it has passed through
the second polarizer?
When the unpolarized light (intensity I0 ) passes through the
rst polarizer, its intensity decreases by half and it will be
polarized in the same direction as the axis of the rst polarizer. Therefore the intensity after the rst polarizer is I1
= I0 /2 = 13 W/cm2 . The light is now polarized, so when
it passes through the second polarizer its intensity decreases
by another factor of cos2 , according to Malus Law:
I2 = I1 cos2 (),
where I2 is the intensity after the second polarizer, and is
the angle between the polarization of the light and the axis
of the polarizer. Since the axis of the rst polarizer is at
24.8 , and the axis of the second polarizer is at 65.0 in the
same direction, the angle between them is the dierence, or
40.2 . Then
I2 = (13 W/cm2 ) cos2 (40.2 ) = 7.58 W/cm2 .
34-58 Saturn is viewed through the Lick Observatory refracting telescope (objective focal length 18 m). If the diameter of the image of Saturn produced by the objective is
1.7 mm, what angle does Saturn subtend from when viewed
from earth?

We now have all the relationships needed to answer the question. We can nd by using the right triangle formed by
the center of the objective lens, the base of the image, and
the tip of the image.
hi
1.7 mm
=
= 9.44 105
fo
18 m
(
)
= tan1 9.44 105 = 9.44 105 rad
tan =

Note that we could have used the small angle approximation, tan . Mastering Physics is expecting an answer in
degrees and to two signicant gures, so we enter 5.4 103 .
35-21 Coherent light with wavelength 450 nm passes
through narrow slits with a separation of 0.350 mm. At a
distance from the slits which is large compared to their separation, what is the phase dierence (in radians) in the light
from the two slits at an angle of 22.6 from the centerline?
Since the light source is coherent, the light that is emitted
from each slit is in phase with each other. When the light
waves arrive at the same point far from the slits, they will
be out of phase by an amount proportional to the dierence
in their path lengths, (r2 r1 ). We call the phase dierence
between these arriving waves . For instance, when the path
dierence is one wavelength, one wave has gone through one
complete cycle more than the other, and = 2 radians.
When the path dierence is /2, = radians, and so on.
We can express this proportionality in the equation
= 2

r2 r1

If the point where the waves meet is far from the slits in
comparison to their separation d, the path dierence is given
by r2 r1 = d sin where is the angle measured from the
centerline. Combining these equations gives
=

2d
2 (0.350 mm)
sin =
sin 22.6 = 1878 rad

450 nm

YF 36-24 An interference pattern is produced by two


identical parallel slits of width a and separation (between
centers) d = 3a. Give the number m of the rst interference
maxima that will be missing in the pattern.
In the gure above (taken from page 1193 in the textbook),
it is shown that a refracting telescope is made of two lenses,
the objective and the eyepiece. Note that in this problem,
we are only dealing with the objective lens. Saturns average
distance from the earth is 1.433 1012 m, which is very large
compared to the other distances in this problem, so we can
use do = . This allows us to calculate di = f = 18 m, or as
shown in the gure, the objective lens creates a real image
at its focal point.
Note that the angle subtended by the object is the same
as the angle subtended by the image. This fact follows by
considering two straight rays: one from the base of the object
to the base of the image, and the other from the tip of the
object to the tip of the image. Both rays pass through the
center of the lens and are not bent; they dene the angles
subtended by the object and by the image. Both angles are
marked as in the gure.

Diraction minima occur when a sin = n, and interference


maxima occur when d sin = m. If these coincide at the
same , then
sin =

m
m
n
m
m
n
=
=

=
n= .
a
d
3a
a
3a
3

The lowest solution for m and n is m = 3 and n = 1.

December 9, 2011

36-9 Sound with a frequency of 1250 Hz leaves a room


through a doorway with a width of 1.00 m. At what minimum angle relative to the centerline perpendicular to the
doorway will someone outside the room hear no sound? Use
344 m/s for the speed of sound in air and assume that the
source and listener are both far enough from the doorway
for Fraunhofer diraction to apply. You can ignore eects of
reections.

K 22-19 The opening to a cave is a tall crack 43.0 cm


wide. A bat that is preparing to leave the cave emits a
31.0 kHz ultrasonic chirp. How wide is the sound beam
110 m outside the cave opening? Use exact formulas; dont
make small angle approximations. Use vsound = 340 m/s.

Even though this example involves sound waves, the basic


phenomenon is still diraction from a single slit. The condition for a dark fringe, or in this case a region with no sound,
is given by
m
sin =
,
a
where m is the order of the region of no sound (m =
1, 2, 3, ...), and a is the width of the door. The wavelength of the sound can be found from the stated frequency
and speed of sound:
=

vsound
344 m/s
=
= 0.2752 m.
f
1250 Hz

To nd the angles, we solve the rst equation above for ,


without making small angle approximations:
m
a
= sin1 (1 0.2752) = 16.0

for m = 1

= sin1 (2 0.2752) = 33.4


= sin1 (3 0.2752) = 55.6

for m = 2
for m = 3

= sin1

Had we used the small angle approximation sin1 , and


converted radians to degrees, we would have obtained 15.8 ,
31.5 , and 47.3 , for m = 1, 2, and 3, respectively. Note
that the maximum possible value of m is 3; otherwise the
sine becomes greater than one.
36-26 A diraction experiment involving two thin parallel
slits yields the pattern of closely spaced bright and dark
fringes shown in the gure. Only the central portion of the
pattern is shown in the gure. The bright spots are equally
spaced at 1.53 mm center to center (except for the missing
spots) on a screen 2.40 m from the slits. The light source
was a He-Ne laser producing a wavelength of 632.8 nm. (a)
How far apart are the two slits? (b) How wide is each one?

36-30 If a diraction grating produces a third-order bright


spot for red light (of wavelength 700 nm) at 67.0 from the
central maximum, at what angle will the second-order bright
spot be for violet light (of wavelength 430 nm)?
(a) The bright spots are caused by constructive interference.
From the optics handout, the condition for constructive interference is
d sin = m
From the information given in the problem, it is possible
to determine d, the slit spacing of the diraction grating.
Setting m = 3 leads to
d=

m
3 700 nm
=
= 2281.4 nm
sin
sin 67.0

Then it is possible to nd the angle for a second-order bright


spot for violet light. Here m = 2, and
(
(
)
)
2 430 nm
m
1
1
= sin
= 22.1
= sin
d
2281.4 nm

Physics 21
Fall, 2011

Information about Exam-1

First Hour Exam: There will be an exam Wednesday,


Oct. 5, 2011 at 4:10 pm in CU248, PA416 and PA466; see the
class web site for room assignments. Extra-time students will
start in CU248, PA416 or PA466 and at 5:10 pm will move to
LL221. The exam will be closed book and closed notes; the
equation sheet posted on the course web site will be included
on the exam. Any physical constants and integrals you need
will be given on the exam.
Use of Calculators and Other Electronic Devices:
You should bring a calculator. Cell phones, music players
and headphones are prohibited and must not even be visible
during the exam. For some questions you may need to solve
three equations in three unknowns. You must solve such
equations by hand and show the solutions for full credit.
You may use a calculator to check your hand solution.
Units: In order to receive full credit, you must include the
correct units with all numerical answers. Be careful about
this point, because the online homework system usually provides the units for you.
Coverage: You are responsible for all the reading assignments, lectures, and homework problems up to and including
R-10, HW-10, and L-10. All reading assigments throught the
end of chapter 27 in the text are covered.
Subject Areas: The emphasis of the exam will be the material covered in lecture or recitation or that has been on the
homework. The following is a list of topics and questions you
should be familiar with. This list is not necessarily complete
but is representative.

What is Gausss Law? Be able to use it to nd the


eld of a spherical, planar, cylindrical, or linear charge
distribution.
What is the electric eld inside a conductor?
In a circuit diagram, what are the symbols for battery,
capacitor, and resistance? Know how to indicate the +
and terminals of a battery.
On a microscopic level, how does a dielectric (insulator)
respond to an electric eld?
What is Ohms Law?
How much power is dissipated when a current passes
through a resistor?
How does a current divide when it branches to ow
through two resistors in parallel?
How do the charges arrange themselves on two capacitors in parallel? in series? What is the eective capacitance for two capacitors in series or parallel? What
about resistors?
What are Kirchhos Rules? Know how to write the
loop and junction (node) equations
that we discussed

in class for any circuit. What is E dl? Know how to
evaluate it for a circuit. Know how to nd the potential
dierence between any two points on an electric circuit.
Know how to write the loop equation for an RC circuit.
What is the time constant for such a circuit? Know
what the function exp(t/RC) looks like. With what
time dependence does charge build up on a capacitor?
With what time dependence does a capacitor discharge?

Know the common prexes femto through Giga.

Know how to verify that a given solution satises the


loop equations for a circuit.

Know the vector form of Coulombs Law. Know how


to nd the electric eld or potential of several point
charges.

Know how to account for the energy stored in the capacitor or lost in the resistor of an RC circuit.

What is the relation between the electric eld and the


electric potential? Know how to evaluate the gradient.
For a distribution of charges on a line, nd . Find
or for two- or three- dimensional distributions.
Know how to integrate over a linear charge distribution
to get the electric eld or potential at an arbitrary point.
Draw the electric eld lines and the equipotential lines
of a point charge. Do the same for a dipole. What is an
equipotential surface?
What is the electric eld of an innite sheet of charge?
Use this result to nd the electric eld between the parallel plates of a capacitor.
What is the energy density in an electric eld? How
much energy is stored in a charged capacitor?

What does it mean to do a surface integral E dA?
What is the convention for the direction of dA if the
surface is closed?

What is current?
What is an electron volt?
What is the dielectric constant? What happens to the
capacitance and electric eld if an insulator with dielectric constant K is placed between the capacitor plates?
Be able to use the right hand rule to get the direction
of magnetic elds or of cross products. Know how to
evaluate cross products.
In a magnetic eld: What is the force on a moving
charged particle? What is the force on a current? How
does a charged particle move in a magnetic eld? How
does a velocity selector work?
What are the total force and torque on a current loop?
What is the magnetic moment?

September 28, 2011

Physics 21
Fall, 2010
1

Hour Exam #1
3

Recitation Time

Total

Recitation Leader

Name:
October 6, 2010

This exam is closed notes and closed book. You must show enough work on each problem to convince the grader
you understand how to solve the problem. Make it easy for the grader to identify your nal answers to each question
or part of a question. You may use a calculator, but show every number that you use to get numerical
results. The penalty for arithmetic errors is small if the grader can tell what you intended to do.
Give units for all nal answers. There is an equation sheet on the last page. All problems count 20 points.
(b) (3 pts.) Determine the currents I1 , I2 , and I3
(including the correct sign) by explicit solution of
the equations you determined in part (a). You
must show your work.

Problem 1. Consider the following circuit:


I2

I3

I1
21 V
3

7
2
11 V

(a) (13 pts.) Write Kirchhos loop and junction (or


node) equations needed to determine the currents
I1 , I2 , and I3 . Use the currents and their directions specied by the arrows in the diagram.
You must write these equations as defined
in Physics 21. You should have three equations
for three unknowns. Draw clearly and label
on the diagram the loop used to determine
each loop equation.

(c) (2 pts.) Use the currents you calculated in part


(b) to nd the potential dierence Vd Va between the corner points labelled d and a on the
diagram, using the path through corner point b.
Show your work.
(d) (2 pts.) Repeat part (c), but take the path
through corner point c. Show your work, including the potential change across the resistor and
battery separately. You should get the same value
for Vd Va that you found in part (c).

Problem 2. Two parallel plate capacitors are connected to a battery and a resistor as shown in the circuit. For the rst capacitor, C1 = 1.24 nF; for the
other, C2 = 3.72 nF. The voltage of the battery is
V0 = 12 volts, and the resistance R = 3500 .
S

(a)
V0

C1

Q1

Q2

C2

V0
R

(b) (6 pts.) At time t = 0 the switch S is ipped


so that the circuit appears as shown in panel (b),
and the capacitors discharge through the resistor.
What is the time constant of the circuit?
(c) (2 pts.) What is the current that starts to ow
in the circuit of panel (b) just after t = 0?

(b)

(a) (8 pts.) Assuming that the circuit in panel (a) has


been connected for a very long time, what are the
charges Q1 and Q2 on each capacitor? What is
the energy stored in each capacitor? Give numerical answers.

C1

C2

(d) (4 pts.) At what time t will the total charge on


the two capacitors diminish to 60% of its initial
value? What fraction of the initial energy will
remain at time t = t ?

Problem 3. For this problem you are to nd the electric eld at the point P on the y axis due to the charge
distribution on the x axis. A total charge of 3Q is uniformly spread out from x = b to x = +2b.
P = (0,y,0)

(b) (3 pts.) On the diagram shown, draw the vector


(r r ) (as dened in class) from the charge dQ
on the rod at x = x to the point P on the y axis.
Write an expression for the vector (r r ) that
indicates its components.
(c) (7 pts.) Write an expression for the electric eld
dE at the point P due to the element of charge
dQ at the point x on the rod. The expression
for dE should be in terms of Q, b, dx , x , y, and
NOT dQ.

dQ
-b

x'

2b

(a) (3 pts.) What is the linear charge density on


the rod?

(d) (7 pts.) Integrate to nd ONLY THE y COMPONENT of the electric eld E at the point P
due to the rod. Your answer should be in terms
of Q, b, and y.

Problem 4. A solid sphere of radius R = 2.00 mm


made of an insulating material has a uniform volume
charge density of = 4.80 1012 C/m3 .

(a) (3 pts.) What is the total charge on the sphere?


(b) (3 pts.) Write Gausss Law.
(c) (7 pts.) Use Gausss Law to nd the magnitude
of the electric eld at a distance r = 1.00 mm
from the center of the sphere. Sketch the Gaussian surface that you use and explain your result.
(d) (7 pts.) Find the magnitude of the electric eld
at the surface of the sphere.

Problem 5. Panel (a) shows a negative ion with mass


m = 1.50 1026 kg and charge Q = 2.00 1019 C
moving along the z axis away from eight identical point
charges with q = 3.25 1019 C. These eight charges
are xed in the xy plane as shown in panel (b); all
eight charges are the same distance b = 0.5 nm from
the origin.
(a) PERSPECTIVE VIEW:

(b) VIEW LOOKING DOWN z AXIS:

z
Q (v = v0 k)
y

The charges are in the xy plane at the


vertices of a regular octagon centered
at the origin. All charges are the same
distance (b) from the origin.
y
q
q
q
b

q x

q
q

q
q

(a) (7 pts.) Write an expression for the potential at


an arbitrary point on the z axis due to the xed
charge distribution in the xy plane. Give your
answer in terms of q, z, b, and standard physical
constants. Hint: Use symmetry.
(b) (6 pts.) At the instant shown in the gure, the
z coordinate of the negative ion is 1.2 nm, and
its speed is v0 = 15.3 km/s. Find the potential
energy, kinetic energy, and total energy of the
ion. Show all work and give numerical answers in
joules.
(c) (7 pts.) The negative ion will slow down and
reach a point z = z  where its speed is zero. Show
explicitly the equation that must be solved to nd
z  , and then solve that equation for z  . Assume
that the particle is constrained to stay on the z
axis.

Physics 21
Fall, 2010

Equation Sheet

speed of light in vacuo


Gravitational constant
Avogadros Number
Boltzmanns constant
charge on electron
free space permittivity
free space permeability
gravitational acceleration

F2 on 1 =

3.00 108 m/s


6.67 1011 N m2 /kg2
6.02 1023 mol1
1.38 1023 J/K
1.60 1019 C
8.85 1012 C2 /(N m2 )
4 107 T m/A
9.807 m/s2

c
G
NA
kB
e
0
0
g

1 q1 q2 (r1 r2 )
40 |r1 r2 |3

; Eplane =
r
20
for  plate
0 capacitor
A
A
Q = CV ; C = 0 K = 
d
d
2
Q
2
Ucap = 12 CV = 12
C
Uind = 12 LI 2
L
V = IR
R=
A
P = IV
P = I 2R
1
20
Q
E=
=
0 A

1 dQ (r r )
40 |r r |3

E = V


V
V
V
= i
+ j
+k
x
y
z
V f Vi =

f

E dl

1 Q
dQ
1
; dV =
40 r
40 |r r |
1
uelec = 12 0 E 2 , umag =
B2
20
V=

Work =

R = mv /(qB)




A B = 


F dl


E dA =

Q
0

d
dt

E dl =




u du
=
a2 + u 2

a2 + u 2

du
1
= tan1
a2 + u 2
a

v=

T /

 
u
a

ln a2 + u

1
2

(T =tension)

v = (347.4 m/s)

T /300

v = f = /k
= 2f

k = 2/

T = 1/f

(T =period)

P  = 12 A2 2 v

1
C

RC time constant = RC
LR time constant = L/R
Q(t) for RLC circuit
Q0 exp(Rt/2L) cos t
2 =

= sin cos

1
R2

LC
4L2

cos sin

E dA

u du
=
a2 + u 2

XR = R, XL = L, XC =

solenoid B = 0 nI
solenoid L = 0 N 2 A/l

C = 2r
C = d
A = r2
A = 4r2
V = 43 r3

circumference of circle
circumference of circle
area of circle
surface area of sphere
volume of sphere

cos(a b) = cos a cos b sin a sin b


du

= ln u + a2 + u2
2
2
a +u

)
2

long wire: B =

eective = 1 + 2

= cos

B dA

d
B dl = 0 I + 0 0
dt

sin(

0 I
2R
center loop: B = 0 I/2R
dQ
I=
I = neAvd
dt
Vs
Ns
Is
Np
=
,
=
Vp
Np
Ip
Ns

m =
1
0
=B
= IA

i =Ci (parallel) or Ri (series):

sin(a b) = sin a cos b cos a sin b

B dA = 0

F = qv B; dF = Idl B
0 Idl (r r )
dB =
4
|r r |3

1
1
1
=
+
eective
1
2

circ. orbit

j

k

Ay Az 
By Bz 

i
Ax
Bx

6.626 1034 J s
1.055 1034 J s
9.11 1031 kg
1.6726 1027 kg
1.6749 1027 kg
1.6605 1027 kg
8.99 109 N m2 /C2

h
h
= h/2
me
mp
mn
u
k

i =Ci (series) or Ri (parallel):

Eline =

F = qE
dE (at r) =

Plancks constant
Plancks constant/(2)
electron rest mass
proton rest mass
neutron rest mass
atomic mass unit
1/(40 )

ab
a+b
sin a + sin b = 2 cos
sin
2
2





du
u
=
(a2 + u2 )3/2
a2 a2 + u 2

du
= ln u
u
2

cos2 d =
0

2D
1 2D
= 2
2
x
v t2
1
S=
(E B)
0

c = 1/ 0 0

c 2
S=
=
B0
0
E0 B0
Erms Brms
=
=
20
0

= h/p

1
2

1
un+1
n+1

du
1
= ln(a + bu)
a + bu
b

1
du = eau
a

ln u du = u ln u u

1
 cE02
2 0

un du =

u du
1
=
(a2 + u2 )3/2
a2 + u 2
au

ax2 + bx + c = 0

b b2 4ac
x=
2a

EBv
xp >
h

sin2 d =
0

KE = p2 /(2M )
(plane wave)

p=
hk

(
h = h/2)

ei = cos + i sin

E=
h = hf

(de Broglie)

2 2
h

= i
h
2M x2
t

August 29, 2010

Physics 21
Fall, 2010

Solution to Hour Exam #1

The graders for the problems were:


1 Jones, 2 Faust, 3 Malenda, 4 and 5 Beels
For questions about the grading, see the grader by Oct. 27.

(b) (3 pts.) Determine the currents I1 , I2 , and I3


(including the correct sign) by explicit solution of
the equations you determined in part (a). You
must show your work.

Problem 1. Consider the following circuit:


I2

I3
I1

loop 1

b
loop 2

21 V
3

7
2
11 V

loop 3

(a) (13 pts.) Write Kirchhos loop and junction (or


node) equations needed to determine the currents
I1 , I2 , and I3 . Use the currents and their directions specied by the arrows in the diagram.
You must write these equations as defined
in Physics 21. You should have three equations
for three unknowns. Draw clearly and label
on the diagram the loop used to determine
each loop equation.

(c) (2 pts.) Use the currents you calculated in part


(b) to nd the potential dierence Vd Va between the corner points labelled d and a on the
diagram, using the path through corner point b.
Show your work.
(d) (2 pts.) Repeat part (c), but take the path
through corner point c. Show your work, including the potential change across the resistor and
battery separately. You should get the same value
for Vd Va that you found in part (c).

Problem 2. Two parallel plate capacitors are connected to a battery and a resistor as shown in the circuit. For the rst capacitor, C1 = 1.24 nF; for the
other, C2 = 3.72 nF. The voltage of the battery is
V0 = 12 volts, and the resistance R = 3500 .
S

(a)
V0

C1

Q1

Q2

C2

V0
R

(b) (6 pts.) At time t = 0 the switch S is ipped


so that the circuit appears as shown in panel (b),
and the capacitors discharge through the resistor.
What is the time constant of the circuit?
(c) (2 pts.) What is the current that starts to ow
in the circuit of panel (b) just after t = 0?

(b)

(a) (8 pts.) Assuming that the circuit in panel (a) has


been connected for a very long time, what are the
charges Q1 and Q2 on each capacitor? What is
the energy stored in each capacitor? Give numerical answers.

C1

C2

(d) (4 pts.) At what time t will the total charge on


the two capacitors diminish to 60% of its initial
value? What fraction of the initial energy will
remain at time t = t ?

Problem 3. For this problem you are to nd the electric eld at the point P on the y axis due to the charge
distribution on the x axis. A total charge of 3Q is uniformly spread out from x = b to x = +2b.
P = (0,y,0)

(b) (3 pts.) On the diagram shown, draw the vector


(r r ) (as dened in class) from the charge dQ
on the rod at x = x to the point P on the y axis.
Write an expression for the vector (r r ) that
indicates its components.
(c) (7 pts.) Write an expression for the electric eld
dE at the point P due to the element of charge
dQ at the point x on the rod. The expression
for dE should be in terms of Q, b, dx , x , y, and
NOT dQ.

r - r'

dQ
-b

x'

2b

(a) (3 pts.) What is the linear charge density on


the rod?

(d) (7 pts.) Integrate to nd ONLY THE y COMPONENT of the electric eld E at the point P
due to the rod. Your answer should be in terms
of Q, b, and y.

Problem 4. A solid sphere of radius R = 2.00 mm


made of an insulating material has a uniform volume
charge density of = 4.80 1012 C/m3 .

R/2

(a) (3 pts.) What is the total charge on the sphere?


(b) (3 pts.) Write Gausss Law.
(c) (7 pts.) Use Gausss Law to nd the magnitude
of the electric eld at a distance r = 1.00 mm
from the center of the sphere. Sketch the Gaussian surface that you use and explain your result.
(d) (7 pts.) Find the magnitude of the electric eld
at the surface of the sphere.

Problem 5. Panel (a) shows a negative ion with mass


m = 1.50 1026 kg and charge Q = 2.00 1019 C
moving along the z axis away from eight identical point
charges with q = 3.25 1019 C. These eight charges
are xed in the xy plane as shown in panel (b); all
eight charges are the same distance b = 0.5 nm from
the origin.
(b) VIEW LOOKING DOWN z AXIS:

(a) PERSPECTIVE VIEW:

z
Q (v = v0 k)
y
b

The charges are in the xy plane at the


vertices of a regular octagon centered
at the origin. All charges are the same
distance (b) from the origin.
y
q
q
q
b

q x

q
q

(a) (7 pts.) Write an expression for the potential at


an arbitrary point on the z axis due to the xed
charge distribution in the xy plane. Give your
answer in terms of q, z, b, and standard physical
constants. Hint: Use symmetry.
(b) (6 pts.) At the instant shown in the gure, the
z coordinate of the negative ion is 1.2 nm, and
its speed is v0 = 15.3 km/s. Find the potential
energy, kinetic energy, and total energy of the
ion. Show all work and give numerical answers in
joules.
(c) (7 pts.) The negative ion will slow down and
reach a point z = z  where its speed is zero. Show
explicitly the equation that must be solved to nd
z  , and then solve that equation for z  . Assume
that the particle is constrained to stay on the z
axis.

Physics 21
Fall, 2011

Solution to Hour Exam #1

The graders for the problems were:


1 Tupa, 2 Faust, 3 Beels, 4 Malenda, 5 Glueckstein
For questions about the grading, see the grader by Oct. 26.

(b) (3 pts.) Determine the currents I1 , I2 , and I3 (including the correct sign) by explicit solution, by
hand, of the equations you determined in part (a).
You must show your work.

Problem 1. Consider the following circuit:


I2

a
8V
7
loop 1

I3

I1
2

12 V

6
loop 2

(a) (13 pts.) Write Kirchhos loop and junction (or


node) equations needed to determine the currents
I1 , I2 , and I3 . Use the currents and their directions specied by the arrows in the diagram.
You must write these equations as defined
in Physics 21. You should have three equations
for three unknowns. Draw clearly and label
on the diagram the loop used to determine
each loop equation.

(c) (2 pts.) Use the currents you calculated in part


(b) to nd the potential dierence Vc Vb between
the corner points labelled c and b on the diagram,
using the path through corner point a. Show your
work, including the potential change across any
circuit elements on this path.

(d) (2 pts.) Repeat part (c), but take the path


through corner point d. Show your work, including the potential change across any circuit elements on this path. You should get the same
value for Vc Vb that you found in part (c).

Problem 2. A point charge q is located at the center


of a spherical cavity of radius rcav inside an insulating,
spherical charged solid:

For this problem, q = 6.1 C, rcav = 3.3 cm, the


charge density in the solid is = 1.5 103 C/m3 ,
and you are to use Gauss Law to calculate the electric
eld inside the solid at a distance r = 5.6 cm from the
center of the cavity.
(a) (3 pts.) Write Gauss Law.
(b) (3 pts.) What is the shape and location of the
Gaussian surface S that you will use? Draw and
label S neatly on the diagram to the left.

r
q

rcav

(c) (6 pts.) What is the total charge enclosed by S?

(d) (6 pts.) Find the magnitude of the electric eld


at distance r = 5.6 cm from the point charge q.
(e) (2 pts.) What is the direction of the electric eld?
Justify your answer.

Problem 3. A battery with V = 12 V is connected


to a circuit with capacitors C1 = 3.0 mF, C2 = 3.5 mF,
and C3 = 2.5 mF as shown.
C2
C1
C3
V

(a) (2 pts.) What is the equivalent (or eective) capacitance of the three capacitors shown?
(b) (12 pts.) Assume the circuit has been connected
for a very long time. Find the charge on each
capacitor and the potential dierence across each
capacitor. Identify the charges as Q1 , Q2 , and
Q3 , and the potential dierences as V1 , V2 , and
V3 . Explain carefully the steps you take to determine your answer.
(c) (6 pts.) Suppose it takes the capacitors 15 s after
the battery is connected to become 99% charged.
What is R?

Problem 4. For this problem you are to nd the electric eld at the point P on the x axis due to the charge
distribution on the y axis. A total charge of Q is uniformly spread out on a thin wire of length L. The lower
end of the wire is at the origin.
y

(b) (4 pts.) On the diagram shown, draw the vectors


r and r that correspond to the eld point (P ) and
the charge point (where dQ is), respectively. Give
the vector (r r ) in terms of its components.
(c) (6 pts.) Write an expression for the electric eld
dE at the point P due to the element of charge
dQ on the wire. Show how to write dQ in terms
of the variable of integration.

charge Q
length L

dQ
(0,y',0)
r'
r

P = (x,0,0)

(a) (3 pts.) What is the linear charge density on


the wire?

(d) (7 pts.) Integrate to nd ONLY THE y COMPONENT of the electric eld E at the point P
due to the wire. Your answer should be in terms
of Q, L, x, and physical constants.

Problem 5. A particle moves in a circular orbit in


a uniform magnetic eld B in the z direction (into
the page). The orbit is conned to the xy plane. The
charge and mass of the particle are q = 3.20 1019 C
and m = 6.75 1026 kg, respectively, and the magnitude of B is B = 0.500 T.

(a) (5 pts.) At some time t0 the particle is at the


point shown on the diagram, and its instantaneous velocity is
v = (3.42 104 m/s) i + (3.08 104 m/s) j.
Give the components of the magnetic force F on
the particle at the time t0 . Draw an arrow on the
diagram that shows the direction of F.
(b) (5 pts.) Find the radius R of the circular orbit.

y
x
+z out of page

(c) (5 pts.) How much time does it take for the particle to make one revolution?
(d) (5 pts.) Through what potential dierence would
the particle have to be accelerated from rest to
acquire the speed it has?

Physics 21
Fall, 2011

Information about Exam 2

Second Hour Exam: There will be an exam on Wednesday, November 9, 2011 at 4:10 pm. The exam will be closed
book and closed notes. Any physical constants and integrals
you need will be given on the exam. The equation sheet
posted on the course web site will be included on the exam.
Here are the room assignments:
Chandler Ullman 248
Packard 466
Packard 416

Glueckstein (all), Beels (9 a.m.)


and all extra time students
Malenda (all), Faust (all)
Tupa (all), Beels (10 a.m.)

Use of Calculators and Other Electronic Devices:


You should bring a calculator. Cell phones, music players
and headphones are prohibited and must not even be visible
during the exam.
Units: In order to receive full credit, all numerical answers
must include the correct units. Be careful about this point,
because the online homework system usually provides the
units for you. You should be familiar with the common
prefixes femto through giga.
Practice Exam and Review Session: A practice exam is
available from the class web site. Some of the problems on
the practice exam are taken from previous exams in Physics
21. If you can work the problems on the practice exam within
the time alloted, you should be well prepared for the real
exam. The solution to the practice exam will be posted
on the web and also discussed in a review session Tuesday,
November 8 at 7:10 pm in LL270. Other review sessions may
also be held; check the class website for the schedule.
Coverage: The exam will cover material starting with our
initial discussion of magnetism (the beginning of Chapter
27). There is some overlap with the material on the magnetic
field covered on the first exam. Specifically, the exam will
cover the lectures numbered 918 inclusive and homeworks
1020 inclusive, as well as all the reading assignments in
Chapters 2731 of the text, except for 31.6 on transformers.
Problem 31-37 on hw20 is about transformers and wont be
on the test.
The exam will emphasize the material covered in lecture,
recitation or homework. The following is a list of topics
and questions you should be familiar with. This list is not
necessarily complete but is representative.
Know the common prefixes femto through giga.

Know why a charged particle can exhibit circular or


spiral motion in a magnetic field.
How does a velocity selector work?
What is the magnetic dipole moment of a current loop?
What are the forces and torques on a current loop in a
uniform magnetic field?
How does a simple dc motor work?
What is Faradays Law? What is Lenzs Law? Know
how to apply them.
What are the magnetic and electric flux? Know how to
evaluate them.
What is hysteresis?
What is ferromagnetism? diamagnetism?
netism? What is a magnetic domain?

paramag-

What is mutual inductance? What is self inductance?


Why does a solenoid exhibit self inductance?
What is Amperes Law (final form)? Know how to use
it. What is the displacement current?
What do transient and steady state mean with respect to dc and ac circuits?
Know how to analyze the transient behavior of an LR
circuit. What do the exponential functions exp(t/ )
and 1 exp(t/ ) look like? What is ?
What is the energy stored in a magnetic field?
Why can there be oscillations in an LC circuit? What
is the resonant frequency? Be able to track where the
energy is during the oscillations. What happens if you
add a resistor to the circuit?
What are Maxwells Equations?
Know how to use phasor diagrams. You should know
the phase relations between current and voltage in the
prototypical ac circuits with only R, L, or C elements
besides the power supply. What does it mean if, for
example, the voltage leads the current across a circuit
element? What is the resonant frequency of an RLC
circuit with an ac power supply?
What is the power delivered to an ac circuit?
In an ac circuit, what is the relation between peak values
and rms values?

In a magnetic field, what is the force on a moving


charged particle? What is the force on a current?
Be able to use the right hand rule to get the direction
of magnetic fields or of cross products. Know how to
evaluate cross products.
What does the magnetic field of a current loop look like?
Be able to use the Biot-Savart Law.

November 3, 2011

Physics 21
Fall, 2010
1

Recitation Time

Hour Exam #2
3

Total

Recitation Leader

Name:
November 10, 2010

This exam is closed notes and closed book. You must show enough work on each problem to convince the grader
you understand how to solve the problem. You may use a calculator, but show every number that you use
to get numerical results. The penalty for arithmetic errors is small if the grader can tell what you
intended to do. Give units for all nal answers. There is an equation sheet on the last page. All problems count
20 points.
Problem 1. For the following circuit:

(b) (3 pts.) Evaluate the peak value of the current I


if the peak voltage supplied by the power supply
is 8.00 V. Show all work.
(c) (3 pts.) Evaluate the phase angle . Does the
voltage lead or lag the current?

(a) (8 pts.) If R = 22.0 , L = 25.0 mH, C =


10.0 F, and = 800.0 rad/s, draw a phasor diagram that is approximately to scale. Include a
phasor for the ac voltage V , and give the length
of each phasor. Draw a mark on your diagram to
indicate the phase angle .

(d) (3 pts.) What is the average power delivered to


the circuit by the power supply?
(e) (3 pts.) What is the time interval between a maximum in voltage across the inductor and the next
maximum in voltage across the power supply?

Problem 2. A small circular wire loop is inside a


larger loop that is connected to the circuit as shown.
The values of the circuit elements are L = 15.0 H,
R = 30.0 , and V0 = 12.0 V. At t = 0 the switch
S is closed to complete the circuit.

(c) (5 pts.) Using the box printed below, carefully


draw a plot that shows the current i(t) in the
large loop as a function of time, starting at t = 0.
Fill in numerical values in seconds and amperes
for several tic marks on the horizontal and vertical axes to set reasonable scales for these axes.
(d) (3 pts.) Is the direction of the induced current in
the small circular wire loop soon after the switch
is closed clockwise or counterclockwise?

(a) (5 pts.) What is the time constant of this circuit?


(b) (5 pts.) What is the current in the large loop a
long time after the switch has been closed?

(e) (2 pts.) Now this experiment is repeated. The


large loop remains the same, but the small wire
inner loop is replaced with one that has half the
diameter. When the switch S is closed, would
the induced emf in the wire loop be smaller or
larger than it was in the rst experiment? Briey
explain your answer.
Draw the graph for part (c) in this box:

Problem 3. A metal bar moves to the left with constant speed v = 8.00 m/s through a uniform magnetic
eld of magnitude B = 1.5 T as shown in the diagram.
The distance between the rails is 0.5 m. The only resistance in the circuit may be taken to be the resistance
R = 24 shown.

(a) (4 pts.) Give Faradays Law (as an equation).


(b) (4 pts.) What is the magnitude of the emf induced in the circuit (before the metal bar hits
the resistor)?
(c) (4 pts.) Is the direction of the current induced in
the circuit clockwise or counterclockwise?
(d) (4 pts.) Calculate the current through the resistor.
(e) (4 pts.) Because of the induced current in the
circuit, the magnetic eld exerts a force on the
moving metal bar. Find the magnitude and direction of that force.

Problem 4. The long wire shown in the diagram lies


in the xy plane and carries a current I = 0.500 A. The
positive z axis points out of the page.

For this problem you are to nd the contribution dB


to the magnetic eld at several points due to the dark
segment dl of the wire centered at the origin. The dark
segment has a length 3.0 mm and makes an angle of
60 with the x axis.
(a) (6 pts.) Write a vector expression for the current
element I dl. (Use the unit vectors i and j.)
(b) (7 pts.) Find dB at x = 0, y = 0, z = 3.0 m
(c) (7 pts.) Find dB at x = 4.2 m, y = 7.3 m, z = 0

Problem 5. For the following circuit, C = 25 F and


L = 32 mH. Just before the switch is closed at time
t = 0, the charge on the capacitor is Q0 = 1.25 mC.

(b) (3 pts.) Give the equation that relates the current i shown on the diagram and the charge q on
the capacitor. (i gives the direction of positive
current ow just after the switch is closed.)
(c) (6 pts.) Write the loop equation for this circuit
and convert it to a dierential equation for the
charge on the capacitor, q(t). Verify that the solution to the dierential equation is
q(t) = Q0 cos t.
(d) (4 pts.) Calculate the value of for this circuit.

(a) (4 pts.) What is the total energy stored in the


circuit before the switch is closed?

(e) (3 pts.) What is the energy stored in the electric


eld of the capacitor at an instant when the magnitude of the magnetic field in the inductor is
60% of its maximum value?

Physics 21
Fall, 2010

Equation Sheet

speed of light in vacuo


Gravitational constant
Avogadros Number
Boltzmanns constant
charge on electron
free space permittivity
free space permeability
gravitational acceleration

F2 on 1 =

3.00 108 m/s


6.67 1011 N m2 /kg2
6.02 1023 mol1
1.38 1023 J/K
1.60 1019 C
8.85 1012 C2 /(N m2 )
4 107 T m/A
9.807 m/s2

c
G
NA
kB
e
0
0
g

1 q1 q2 (r1 r2 )
40 |r1 r2 |3

; Eplane =
r
20
for  plate
0 capacitor
A
A
Q = CV ; C = 0 K = 
d
d
2
Q
2
Ucap = 12 CV = 12
C
Uind = 12 LI 2
L
V = IR
R=
A
P = IV
P = I 2R
1
20
Q
E=
=
0 A

1 dQ (r r )
40 |r r |3

E = V


V
V
V
= i
+ j
+k
x
y
z
V f Vi =

f

E dl

1 Q
dQ
1
; dV =
40 r
40 |r r |
1
uelec = 12 0 E 2 , umag =
B2
20
V=

Work =

R = mv /(qB)




A B = 


F dl


E dA =

Q
0

d
dt

E dl =




u du
=
a2 + u 2

a2 + u 2

du
1
= tan1
a2 + u 2
a

v=

T /

 
u
a

ln a2 + u

1
2

(T =tension)

v = (347.4 m/s)

T /300

v = f = /k
= 2f

k = 2/

T = 1/f

(T =period)

P  = 12 A2 2 v

1
C

RC time constant = RC
LR time constant = L/R
Q(t) for RLC circuit
Q0 exp(Rt/2L) cos t
2 =

= sin cos

1
R2

LC
4L2

cos sin

E dA

u du
=
a2 + u 2

XR = R, XL = L, XC =

solenoid B = 0 nI
solenoid L = 0 N 2 A/l

C = 2r
C = d
A = r2
A = 4r2
V = 43 r3

circumference of circle
circumference of circle
area of circle
surface area of sphere
volume of sphere

cos(a b) = cos a cos b sin a sin b


du

= ln u + a2 + u2
2
2
a +u

)
2

long wire: B =

eective = 1 + 2

= cos

B dA

d
B dl = 0 I + 0 0
dt

sin(

0 I
2R
center loop: B = 0 I/2R
dQ
I=
I = neAvd
dt
Vs
Ns
Is
Np
=
,
=
Vp
Np
Ip
Ns

m =
1
0
=B
= IA

i =Ci (parallel) or Ri (series):

sin(a b) = sin a cos b cos a sin b

B dA = 0

F = qv B; dF = Idl B
0 Idl (r r )
dB =
4
|r r |3

1
1
1
=
+
eective
1
2

circ. orbit

j

k

Ay Az 
By Bz 

i
Ax
Bx

6.626 1034 J s
1.055 1034 J s
9.11 1031 kg
1.6726 1027 kg
1.6749 1027 kg
1.6605 1027 kg
8.99 109 N m2 /C2

h
h
= h/2
me
mp
mn
u
k

i =Ci (series) or Ri (parallel):

Eline =

F = qE
dE (at r) =

Plancks constant
Plancks constant/(2)
electron rest mass
proton rest mass
neutron rest mass
atomic mass unit
1/(40 )

ab
a+b
sin a + sin b = 2 cos
sin
2
2





du
u
=
(a2 + u2 )3/2
a2 a2 + u 2

du
= ln u
u
2

cos2 d =
0

2D
1 2D
= 2
2
x
v t2
1
S=
(E B)
0

c = 1/ 0 0

c 2
S=
=
B0
0
E0 B0
Erms Brms
=
=
20
0

= h/p

1
2

1
un+1
n+1

du
1
= ln(a + bu)
a + bu
b

1
du = eau
a

ln u du = u ln u u

1
 cE02
2 0

un du =

u du
1
=
(a2 + u2 )3/2
a2 + u 2
au

ax2 + bx + c = 0

b b2 4ac
x=
2a

EBv
xp >
h

sin2 d =
0

KE = p2 /(2M )
(plane wave)

p=
hk

(
h = h/2)

ei = cos + i sin

E=
h = hf

(de Broglie)

2 2
h

= i
h
2M x2
t

August 29, 2010

Physics 21
Fall, 2010

Solution to Hour Exam #2

If you want to discuss the grading, you must speak with the grader by Dec. 8.
1: Beels 2: Jones 3: Faust 4: Beels 5: Glueckstein
Problem 1. For the following circuit:

(b) (3 pts.) Evaluate the peak value of the current I


if the peak voltage supplied by the power supply
is 8.00 V. Show all work.
(c) (3 pts.) Evaluate the phase angle . Does the
voltage lead or lag the current?

(a) (8 pts.) If R = 22.0 , L = 25.0 mH, C =


10.0 F, and = 800.0 rad/s, draw a phasor diagram that is approximately to scale. Include a
phasor for the ac voltage V , and give the length
of each phasor. Draw a mark on your diagram to
indicate the phase angle .

(d) (3 pts.) What is the average power delivered to


the circuit by the power supply?
(e) (3 pts.) What is the time interval between a maximum in voltage across the inductor and the next
maximum in voltage across the power supply?

Problem 2. A small circular wire loop is inside a


larger loop that is connected to the circuit as shown.
The values of the circuit elements are L = 15.0 H,
R = 30.0 , and V0 = 12.0 V. At t = 0 the switch
S is closed to complete the circuit.

(c) (5 pts.) Using the box printed below, carefully


draw a plot that shows the current i(t) in the
large loop as a function of time, starting at t = 0.
Fill in numerical values in seconds and amperes
for several tic marks on the horizontal and vertical axes to set reasonable scales for these axes.
(d) (3 pts.) Is the direction of the induced current in
the small circular wire loop soon after the switch
is closed clockwise or counterclockwise?

(a) (5 pts.) What is the time constant of this circuit?


(b) (5 pts.) What is the current in the large loop a
long time after the switch has been closed?

(e) (2 pts.) Now this experiment is repeated. The


large loop remains the same, but the small wire
inner loop is replaced with one that has half the
diameter. When the switch S is closed, would
the induced emf in the wire loop be smaller or
larger than it was in the rst experiment? Briey
explain your answer.
Draw the graph for part (c) in this box:

Problem 3. A metal bar moves to the left with constant speed v = 8.00 m/s through a uniform magnetic
eld of magnitude B = 1.5 T as shown in the diagram.
The distance between the rails is 0.5 m. The only resistance in the circuit may be taken to be the resistance
R = 24 shown.

(a) (4 pts.) Give Faradays Law (as an equation).


(b) (4 pts.) What is the magnitude of the emf induced in the circuit (before the metal bar hits
the resistor)?
(c) (4 pts.) Is the direction of the current induced in
the circuit clockwise or counterclockwise?
(d) (4 pts.) Calculate the current through the resistor.
(e) (4 pts.) Because of the induced current in the
circuit, the magnetic eld exerts a force on the
moving metal bar. Find the magnitude and direction of that force.

Problem 4. The long wire shown in the diagram lies


in the xy plane and carries a current I = 0.500 A. The
positive z axis points out of the page.

For this problem you are to nd the contribution dB


to the magnetic eld at several points due to the dark
segment dl of the wire centered at the origin. The dark
segment has a length 3.0 mm and makes an angle of
60 with the x axis.
(a) (6 pts.) Write a vector expression for the current
element I dl. (Use the unit vectors i and j.)
(b) (7 pts.) Find dB at x = 0, y = 0, z = 3.0 m
(c) (7 pts.) Find dB at x = 4.2 m, y = 7.3 m, z = 0

Problem 5. For the following circuit, C = 25 F and


L = 32 mH. Just before the switch is closed at time
t = 0, the charge on the capacitor is Q0 = 1.25 mC.

(b) (3 pts.) Give the equation that relates the current i shown on the diagram and the charge q on
the capacitor. (i gives the direction of positive
current ow just after the switch is closed.)
(c) (6 pts.) Write the loop equation for this circuit
and convert it to a dierential equation for the
charge on the capacitor, q(t). Verify that the solution to the dierential equation is
q(t) = Q0 cos t.
(d) (4 pts.) Calculate the value of for this circuit.

(a) (4 pts.) What is the total energy stored in the


circuit before the switch is closed?

(e) (3 pts.) What is the energy stored in the electric


eld of the capacitor at an instant when the magnitude of the magnetic field in the inductor is
60% of its maximum value?

Physics 21
Fall, 2011

Information about Final

What is refraction? What is the index of refraction?


What is disperson?

Final Exam: The nal exam will be Thursday, December


15, 2011 from 7:1010:10 pm. Almost everyone will be in
PA 101; extra time students will be in LL221 and will start
90 minutes early, at 5:30 pm. The exam will be closed book
and closed notes. The equation sheet and the handout on
optics (now posted on the course web site) will be included
with the exam. Any physical constants and integrals you
will need will be given on the equation sheet.

Know the sign conventions for converging and diverging lenses. Know how to use the lensmakers equation,
including the sign conventions for the radii of curvature.

Practice Questions: Representative questions on waves


and optics taken from previous exams in Physics 21 will be
posted on the class web site. The two previous hour exams and the associated practice exams provide representative questions on electricity and magnetism. The solution to
the practice questions will be posted on the web.

What is total internal reection? What is the critical


angle?

Use of Calculators: You should bring a calculator. In


general, however, setting up the problems and demonstrating
the correct strategy for solving them are worth more than
doing the arithmetic to get a numerical result. If you are
asked to solve simultaneous algebraic equations, you must
solve them by hand and show the solution to receive full
credit.
Physics 19: The Physics 19 exam will consist of selected
problems from the full Physics 21 exam. Physics 19 students
may take the full three hours to work the exam.
Coverage: The exam will cover all the material presented
this semester. About 50% of the exam will be on electricity
and magnetism and about 50% on waves and optics. The
emphasis will be on the material covered in lecture, recitation or homework. Some questions may come from material
presented only in the lectures. There may be some shortanswer questions, that is, you might be asked for a short
denition or an example. You will not be asked to solve differential equations, but you may be asked to verify that a
given function satises a dierential equation.
You should be familiar with all the topics and questions
listed on the study guides for Hour Exams #1 and #2, as
well as with the items listed below that cover optics. The
list is not necessarily complete but is representative.
What is the Poynting vector? Know how to use it to
describe the transport of energy by an E&M wave.
What is the dierence between geometric and physical
optics? What is the essential assumption of geometric
optics?

How can you describe the path of a light ray using the
principle of least time?
Why is the case of parallel light rays important for a
lens or mirror?

What makes a rainbow?


What is the mathematical form of a traveling plane
wave?
Know how to determine where the sound from two
speakers will interfere constructively or destructively.
How does your analysis depend on whether the speakers
are in phase or out of phase?
What is chromatic aberration?
What is an achromatic doublet?
Be able to give examples of and explain phenomena included under the heading of diraction and interference.
What is a diraction grating?
Be able to sketch the interference pattern for the two slit
experiment if you are given the width and separation of
the slits. What are missing orders and why do they
occur?
What are the approximate wavelengths of visible light?
What is Huygens principle?
What is the f -number of a lens?
What is the dierence between a real and a virtual image?
What is the dierence between discrete and continuous
spectra? Be able to give an example of each.
What limits the ability of a telescope to resolve two
closely spaced binary stars? What is the Rayleigh criterion?

What is the law of reection (for a mirror)?

What determines the polarization of an E&M wave?


What is the relation between E and B and the velocity
of the wave?

Know how to use Snells Law.

What does a polarizing lter do?

What is the paraxial approximation?


Be able to draw ray diagams for mirrors and lenses using
the notes handed out. Be able to draw a ray diagram for
a magnifying glass or a telescope. Be able to analyize a
two-lens system algebraically.

December 9, 2011

Physics 21
Fall, 2011

Practice Questions on Optics

Some of these questions on optics have been adapted from questions on the Physics 21 nal given at the end of
the spring semester of 2004. Try to work them using only the equation sheet and the notes on optics that will be
provided with the nal.
Problem 1. Consider the polarizers in this problem
as perfect. After passing through the rst polarizer,
the electric eld of a light wave is (in SI units)
E = E0 cos[6.0 1015 t 2.0 107 x]j,
with E0 = 2.0 104 V/m.

Problem 3. A diverging lens with a focal length f =


0.06 m is 0.14 m from an object of height 0.02 m.
(a) Algebraically determine the location and the height
of the image.
(b) Determine the location and the height of the image using a ray diagram.

(a) What are the frequency and wavelength of this


wave?
(b) A second polarizer is oriented at 60 to the rst.
What is the magnitude and direction of the electric eld after passing through this second polarizer?
y

(c) Is the image real or virtual?


Problem 4. At noon on the rst day of spring, the
sun is directly overhead at a point on the equator; i.e.,
the suns shines perpendicularly to the earths surface
at that point. Let z be the direction straight down.
Assume the electric and magnetic elds of this wave
are (in SI units)


60O
z

(c) A third polarizer is oriented in the z direction.


What is the magnitude and direction of the electric eld after passing through the third polarizer?
Problem 2. Two slits that are d = 2.0 104 m
apart are illuminated with monochromatic light with a
wavelength of 600 nm.

L
(a) What is the angular separation between the central fringe and the next bright fringe?
(b) What is the spacing (in meters) between the bright
fringes if the screen is L = 3.0 m from the slits?



z
i
E(z, t) = E0 cos 2 5.0 10 t
6.0 107
 

z
14
j
B(z, t) = B0 cos 2 5.0 10 t
6.0 107
14

(a) What are the frequency, wavelength and velocity


of this wave?
(b) If E0 = 1000 V/m, what is the average intensity
(power/meter2 ) of this sunshine?
Problem 5. Two stars 10 light years away are barely
resolved by a 90 cm (mirror diameter) telescope. How
far apart are the stars? Assume = 550 nm and that
the resolution is limited by diraction.

Physics 21
Fall, 2011

Solution to Practice Questions

Some of these questions on optics have been adapted from questions on the Physics 21 nal given at the end of
the spring semester of 2004. Try to work them using only the equation sheet and the notes on optics that will be
provided with the nal.
Problem 1. Consider the polarizers in this problem
as perfect. After passing through the rst polarizer,
the electric eld of a light wave is (in SI units)

Problem 2. Two slits that are d = 2.0 104 m


apart are illuminated with monochromatic light with a
wavelength of 600 nm.

E = E0 cos[6.0 1015 t 2.0 107 x]j,


with E0 = 2.0 104 V/m.
(a) What are the frequency and wavelength of this
wave?
= 2f = 6.0 1015 f = 9.5 1014 Hz
k = 2/ = 2.0 107 = 3.1 107 m
(b) A second polarizer is oriented at 60 to the rst.
What is the magnitude and direction of the electric eld after passing through this second polarizer?

E0

L
(a) What is the angular separation between the central fringe and the next bright fringe?
The angular splitting (in radians) is given by

60
E1
E2

30O

The projection of E0 on the axis of the second


polarizer is E1 = E0 cos 60 = 1 104 V/m.
(c) A third polarizer is oriented in the z direction.
What is the magnitude and direction of the electric eld after passing through the third polarizer?
The projection of E1 on the axis of the third polarizer is E2 = E1 cos 30 = 8.7 103 V/m.

600 109

=
= 0.003
d
2 104

Note that this angular splitting is small enough


that the small angle approximations used are justied.
(b) What is the spacing (in meters) between the bright
fringes if the screen is L = 3.0 m from the slits?
The spacing in meters is L = 3.0 m 0.003 =
0.009 m = 9 mm.

Problem 3. A diverging lens with a focal length f =


0.06 m is 0.14 m from an object of height 0.02 m.
(a) Algebraically determine the location and the height
of the image.
Lets work the problem in cm. We have do = 14
and f = 6. The lens equation gives
1
1
1
1
1
1

=
=
=
di
f
do
6 14
4.2
So the image is 4.2 cm in front of the lens. The
height is
hi =

4.2
2 = 0.6 cm
14

(b) Determine the location and the height of the image using a ray diagram.

Problem 4. At noon on the rst day of spring, the


sun is directly overhead at a point on the equator; i.e.,
the suns shines perpendicularly to the earths surface
at that point. Let z be the direction straight down.
Assume the electric and magnetic elds of this wave
are (in SI units)




z
i
E(z, t) = E0 cos 2 5.0 10 t
6.0 107
 

z
14
j
B(z, t) = B0 cos 2 5.0 10 t
6.0 107
14

(a) What are the frequency, wavelength and velocity


of this wave?

= 2f = 2(5.0 1014 ) f = 5 1014 Hz


2
= 6 107 m
k = 2/ =
6.0 107

1
object

F
image

2
3

(The diagram above is not exactly to scale, but


its close enough to show how the ray diagram
looks.)
(c) Is the image real or virtual?
The image cannot be focussed on a screen, so it
is virtual.

(b) If E0 = 1000 V/m, what is the average intensity


(power/meter2 ) of this sunshine?
The expression that uses only the electric eld amplitude E0 is
S = 12 0 cE02 = .5(8.85 1012 )(3.00 108 )(1000)2
= 1330 W/m2

Using the fact that B0 = E0 /c for a plane wave, one


can write equivalent formulas involving just E0 or B0
that give the same result.
Problem 5. Two stars 10 light years away are barely
resolved by a 90 cm (mirror diameter) telescope. How
far apart are the stars? Assume = 550 nm and that
the resolution is limited by diraction.
The Rayleigh criterion gives
= 1.22

550 109 m
= 1.22
= 7.46 107 rad
D
0.9 m

The absolute distance is given by L, where L is


10 ly. That works out to 7 1010 m, which is about 4
light minutes, or about half the distance from the earth
to the sun.

Das könnte Ihnen auch gefallen